You are on page 1of 25

UST MEDICINE – OTORHINOLARYNGOLOGY ORAL REVALIDA ΣΒΤΦ TITANS

THE HEAD & NECK HISTORY AND PHYSICAL EXAMINATION


OTORHINOLAYNGOLOGY CONSULTATION FORM

GENERAL HEAD & NECK EXAM ■ Mouth & Pharynx


(1) shine the light then let patient extrude the tongue
■ Head & Face (2) have the patient move his tongue up, to the sides, and protrude → inspect
(3) use tongue depressor to depress the RELAXED tongue → inspect posterior, elicit gag, cheek tenderness

● Inspection
○ head: normocephalic, no head tilting or any abnormal head or facial movement ● Inspection of the Oral Cavity
○ scalp: no depressions, prominence or swelling ○ mucosal dryness, lesions & masses
○ face: no characteristic facies, no facial asymmetry, dysmorphic features, lumps ○ tongue lesion, masses, mobility
○ check for dental caries
● Palpation ○ ductal orifices: erythema? pus? turbid discharge? lack of discharge? evident sialolith?
○ scalp non-tender ○ deviation of the tongue and uvula
○ tonsils: presence/absence, size, surface color, exudates, tonsilloliths, adherent membranes
● Auscultation ○ posterior pharyngeal wall: color? post-nasal drip? cobblestoning?
○ temporal artery non-tender with no bruit
○ no bruit in the eye or occipital area ● Palpation
○ check for cheek tenderness
■ Ears ○ gag reflex intact
● Inspection (External Ear) ○ if suspected, may perform tongue base palpation for masses
○ size, shape, location (low set?), symmetry
○ discharge: serous, purulent, blood-stained ■ Larynx
○ appearance: erythema? swelling? skin lesions? masses?
● Indirect Laryngoscopy
*inverted image through a mirror: inverted V, left is right and right is left
● Palpation (External Ear) (1) patient leaning forward, chin pointing upward (sniffing position); wear gloves, pull the tongue forwards using gauze
○ palpate preauricular, postauricular, mastoid areas (2) insert the laryngeal mirror pointing downwards beneath palate & uvula, push up & back
(3) check 2 positions of the larynx
○ tragal tenderness, tenderness on traction of the pinna □ Phonation Position: tell patient to say “hee”
□ Respiratory Position: tell patient to take a deep breath

● Tests for Hearing Loss ○ vocal fold: color (pearly white), movement (no restrictions)
○ mucosa: surface (no ulcerations), edge (not irregular)
● Clinical Hearing Tests ○ arytenoid, posterior commissure – non-hyperemic, no hypertrophy
○ Finger Rub Test: (+) of failure to identify ≥3/6 attempts
□ rub 2 fingers together 6 inches away ■ Neck
○ Whispered Voice Test: (+) of failure to identify ≥3/6 attempts
□ occlude non-tested ear; instruct the patient to repeat letter/number combination heard ● Inspection
□ whisper letter/number combinations at arm’s length away ○ symmetry, muscle tone & bulk, neck veins

● Tuning Fork Tests ● Palpation


○ Rinne Test: AC>BC (normal or SNHL); BC>AC (conductive) ○ palpate the thyroid, then swallow
○ Weber Test: lateralization (to normal ear – SNHL, to involved ear – conductive) ○ palpate for cervical lymph nodes
○ palpate for the carotid pulse – report in CARDIAC PE
● Otoscopy
(1) using the largest otoscope: enable to see the entire TM ● CN XI Exam
(2) pull ear upwards and backwards (adults), downwards and backwards (children) ○ lift shoulder, turn head to the sides – report in NEURO
(3) hold the otoscope like a pen; extend the 5th digit to stabilize the otoscope to the head
(4) insert the otoscope, but stay only in the cartilaginous outer 1/3
○ ear canal: discharge, color, crusting/scaling, lumen narrowing ● Auscultation
○ tympanic membrane ○ auscultate the thyroid – check for bruits
□ color: pearly gray, translucent
□ intact, differentiated (distinguishable fibrocartilaginous ring & handle of malleus) ● JVP measurement
□ if perforated, state location and size ○ done in supine position
□ positive cone of light (there is loss of cone if swollen) *done with cardiac and abdominal exam
□ mobility: mobile, restricted – can be done via pneumatic otoscopy, Valsalva, Tonybee

■ Nose

● Inspection (External Nose)


○ check for patency, alar flaring, discharge, position of septum, gross deformities

● Anterior Rhinoscopy
(1) hold nasal speculum; thumb on the fulcrum, index finger on the nose to stabilize
(2) insert the speculum closed, and then gently open
(3) retract the nasal speculum, semi-open – don’t fully close it, you might pull some vibrissae
○ discharge? no discharge
○ color of the mucosa? non-hyperemic
○ turbinates? color, swelling
○ mass? color, size, shape, mobility, pedunculated/sessile

● Posterior Rhinoscopy
*not done in all patients, not done in non-compliant patients
*indications: suspected mass lesions, anatomic anomaly
(1) use tongue depressor to depress the tongue
(2) insert the nasopharyngeal (smaller) mirror pointing upwards; side to side to visualize
□ avoid gag reflex by avoiding: posterior pharyngeal wall, base of tongue
○ findings: state similarly as anterior rhinoscopy

LACABA 1|PAGE
UST MEDICINE – OTORHINOLARYNGOLOGY ORAL REVALIDA ΣΒΤΦ TITANS

OTOLOGY
Approach to OTOLOGIC CHIEF COMPLAINTS
CC: OTALGIA ■ CC, HPI, ROS CC: OTORRHEA ■ General Data
■ Common Differential Diagnosis for REVALIDA (for exhaustive differentials, refer to AAFP 2018) ■ Common Differential Diagnosis
○ Otitis Externa: Circumscribed, Diffuse, Necrotizing ● Otalgia *in a patient presenting with otorrhea, consider conditions with TM perforations as one of your differentials ● Age – presbycusis
○ Acute Otitis Media, Chronic Otitis Media ○ laterality ● Occupation – noise-induced hearing loss
○ Ear Trauma (Barotrauma) ○ OLDCARTS of pain (otalgia) ● Acute Otorrhea (≤6 weeks)
○ Foreign Object in the ear □ Localization - ask the patient to localize the pain, this may reveal a secondary cause ○ Otitis Externa ■ CC, HPI, ROS
○ Temporomandibular Dysfunction *localization to the TMJ → TMJ Syndrome ○ Acute Otitis Media – only if with spontaneous TM rupture
*mastoid pain → Myofascial pain, Mastoiditis ○ Postoperative Complication of Tympanostomy tube ● Hearing Loss
*preauricular pain → possible salivary gland disorder ○ self-recognized or concerns from family members
□ Timing ● Chronic Otorrhea (>6 weeks) □ difficulty understanding everyday conversation, frequently ask others to repeat something
*continuous & progressive pain → likely infection & primary otalgia ○ Chronic Suppurative Otitis Media – most common cause □ turning up TV volume, difficulty hearing with background noise
*intermittent pain → likely secondary otalgia ○ Cholesteatoma – only if infected cholesteatoma ○ laterality
*episodic pain lasting seconds → neuralgia ○ Foreign Body □ unilateral or asymmetric → acoustic trauma, external & middle ear causes
○ Others: Granuloma, Immunodeficiency, Neoplasm □ bilateral → otosclerosis
*have the patient describe the pain and rule out MI, thoracic aneurysms □ progressive, bilateral, symmetric → NIHL, presbycusis, ototoxicity SSNHL
■ History ○ onset/duration, pain
● Associated symptoms suggesting PRIMARY OTALGIA ○ laterality □ sudden/gradual painless → impacted cerumen, foreign body
○ hearing loss/tinnitus → if absent, think non-otologic disease ○ onset/duration □ sudden painful → otitis externa, AOM, COM
○ ear fullness ○ consistency (serous, bloody, purulent) □ gradual painless → OME, cholesteatoma, otosclerosis
○ ear discharge → think TM perforation esp. if mucopurulent ○ otalgia → proceed as mentioned in CC: OTALGIA!
○ hearing loss/tinnitus ● Tinnitus – commonly accompanies hearing loss in adults; consider the following:
● Associated symptoms suggesting SECONDARY OTALGIA ○ history of swimming ○ abnormal otoscopy → impacted cerumen, otitis media
○ pain with chewing, neck movement → myofascial pain? ○ normal otoscopy → NIHL, acoustic trauma, ototoxicity, Meniere’s disease, semicircular canal dehiscence
○ sinusitis (HPI)
○ PMHx: Hx dental procedures, Hx GERD CC: HEARING LOSS ● Rule In: Causes of Conductive Hearing Loss
■ Common Differential Diagnosis for REVALIDA (for exhaustive differentials, refer to AAFP 2003, 2019) ○ Symptoms
● Other Associated Symptoms ● Conductive Hearing Loss □ ear fullness → OME
○ Hx nasal congestion, URTI → middle ear infections eg AOM *if HL is conductive, check for otalgia and otorrhea, proceed with HxPE based on those symptoms! ○ Possible Causes
○ history of trauma/manipulation → ear trauma ○ External Ear Causes □ history of trauma → TM perforation, ossicular disruption
○ history of swimming → diffuse otitis externa □ Impacted Cerumen □ PMHx: recurrent otitis media → OME, ossicular disruption
□ Foreign Body □ history of URTI → OME
■ Tertiary History □ Otitis Externa □ history of swimming → diffuse otitis externa
○ Middle Ear Causes □ ear manipulation → otitis externa, impacted cerumen
● Risk Factors for Head & Neck Tumors □ TM Perforation secondary to AOM, COM, Trauma
○ smoking □ COM – most common cause of HL in children ● Rule In: Causes of Sensorineural Hearing Loss
○ alcohol use □ Cholesteatoma ○ Symptoms
○ weight loss □ Otitis Media with Effusion (OME) □ dizziness/vertigo → check Meniere’s disease, Acoustic Neuroma
PRIMARY OTALGIA: COMMON CAUSES ○ previous radiation exposure ○ Possible Causes
CONDUCTIVE HEARING LOSS □ history of URTI → labyrinthitis
Etiology History Physical Examination
Etiology History Physical Examination
Otitis Externa recent swimming, history of otorrhea tragal & auricular tenderness, (±) otorrhea ■ Physical Examination □ PMHx meningitis, recurrent otitis media → labyrinthitis
EXTERNAL EAR
Otitis Media recent URTI, children may pull on ears red, inflamed, cloudy TM
Impacted Cerumen ○ HL: gradual, painless occlusive cerumen
□ Medication Hx: aminoglycosides, loop diuretics, quinine, salicylates, cisplatin → ototoxicity
Barotrauma
pain starts with scuba diving/in airplane; recent
(±) middle ear hemorrhage ● Vital Signs Foreign Body ○ HL: gradual, painless foreign body in ear canal □ history of exposure to loud noises → noise-induced hearing loss
blast injury ○ unstable vital signs → MI or thoracic aneurysm with referred otalgia ○ HL: sudden, painful (otalgia) narrow ear canal; dry cracked erythematous
Foreign Body insects, small toys, peanuts; m/c children foreign object in ear canal Otitis Externa
○ Hx swimming, otorrhea walls (diffuse); canal swelling (circumscribed) ■ Physical Examination
ET Dysfunction Sx of pressure dysregulation (ear fullness) TM retraction, tympanometry Type C
● Ear Examination TYMPANIC MEMBRANE
○ Inspection of the auricle & preauricular region TM Perforation *may be secondary to AOM, COM, Trauma ● Vital Signs
PRIMARY OTALGIA: UNCOMMON CAUSES AOM ○ recent URTI, otalgia, ± otorrhea
Etiology History Physical Examination □ erythema → Cellulitis *assess if patient is stable, very important if considering otogenic meningitis
COM ○ chronic otorrhea normal canal; red, immobile TM
Cellulitis of Auricle preceding bite, scratch, piercing earlobe usually involved □ bleeding → Ear trauma ○ ossicular injuries, hemotympanum
○ fever → meningitis
Cholesteatoma ear fullness pearly mass visible through TM □ otorrhea → fetid discharge (Necrotizing OE) Trauma ○ bradycardia, ↑pulse pressure (↑SBP ↓DBP), irregular respiration → Cushing’s Triad in ↑ICP
○ Hx: barotrauma, head trauma
Malignant
retroauricular pain, PMHx DM,
granulation tissue may be present
□ vesicles → Ramsay Hunt Syndrome Tympanosclerosis
immunocompromised state; HPI previous OE ○ Otoscopic Findings: Ear Canal MIDDLE EAR ● Ear Examination
(Necrotizing) OE (±) lower CN neuropathies
with no improvement
□ Circumscribed OE: ear canal swelling with residual lumen Cholesteatoma ○ HL: gradual onset, painless retracted or perforated TM, otorrhea if infected *see approach to otalgia for assessment of external & middle ear causes of HL
Mastoiditis recent COM, may be AOM tender, edematous mastoids Ossicular Chain
Herpes Zoster pain (may be present even before lesions) vesicular rash on auricle □ Diffuse OE: dry cracked, scaly/crusting, diffuse swelling ○ Hx: Trauma, recurrent Otitis Media ○ Inspection & Palpation of the External Ear
Disruption
Oticus ± hearing loss, vertigo, tinnitus ± facial nerve palsy □ Necrotizing OE: ulcer on canal floor, with exposed brownish bone, fetid discharge opaque amber-colored, immobile TM; (±) air
○ Otoscopy: ear canal; TM retraction/perforation, cholesteatoma, immobility
Trauma blunt trauma, burns evidence of trauma (m/c laceration) □ look for foreign body in the ear OME ○ recent URTI, ear fullness, popping/sloshing
bubbles if incomplete OME ○ Clinical Examination
Viral Myringitis similar to AOM (with recent URTI)
TM red but not bulging ○ Otoscopic Findings: Tympanic Membrane ○ gradual, painless □ Finger Rub Test: failure to identify rub 6 inches away (≥3/6 attempts)
hemorrhagic bullae of TM or ear canals □ AOM: middle ear effusion, moderate-severe bulging TM, erythema Otosclerosis ○ bilateral HL 30-50yo normal mobile TM □ Whispered Voice Test: inability to repeat number/letter combo from an arm’s length away (≥3/6 attempts)
*immobile on pneumatic otoscopy, don’t do Valsalva maneuver! ○ better at hearing in noisy environment □ Speech Test: normal/raised voice 1 meter away
SECONDARY OTALGIA: SUBACUTE ETIOLOGIES □ Viral Myringitis: erythematous but NOT bulging TM, hemorrhagic bullae in the ear canal ○ Tuning Fork Tests (512Hz)
Etiology History Physical Examination ● Sensorineural Hearing Loss
○ Palpation □ Weber Test: lateralize to affected ear (CHL); lateralize to normal ear (SNHL)
Dental Causes dental symptoms Caries; abscess; gingivitis; facial swelling ○ Noise-induced Hearing Loss
(Caries, Abscess, Pulpitis) □ check for tragal tenderness, traction of auricles → OE □ Rinne Test: AC>BC (SNHL, normal); BC>AC (CHL)
GER □ palpate mastoids → mastoiditis, trigger point for myofascial pain ○ Ototoxic Drugs ○ Objective Tests
acid reflux unremarkable
Irritation on CN 9, 10 ○ Presbycusis □ Audiometry
Increased risk: smoking, alcohol use, ≥50yo, ○ Sudden Sensorineural Hearing Loss
Head & Neck radiation exposure, weight loss possible painless neck mass, consider *if the auricular exam is unremarkable, a complete Head & Neck examination should also be performed *CHL: abnormal AC, normal BC
Tumors pain worse with swallowing, especially acidic or fiberoptic nasolaryngoscopy *pay special attention to the TMJ and the nasal and oral cavities *SNHL: abnormal AC, abnormal BC, (-) air bone gap
spicy foods ● SNHL with Vestibular Symptoms *Mixed: abnormal AC, abnormal BC, (+) air bone gap
Myofascial Pain
cervical pain (may be aggravated by chewing, Likely to have trigger point: neck or mastoid tip ● HENT Examination ○ Labyrinthitis, Meniere’s Disease, Acoustic Neuroma □ Tympanometry
neck movement) at attachment of SCM
Neuralgia
○ Head, Scalp, Face *Type As (low compliance): stiff ossicles (otosclerosis)
may have trigger point, but examination is □ vesicles → Ramsay Hunt Syndrome SENSORINEURAL HEARING LOSS *Type Ad (high compliance): disarticulated ossicles (trauma, recurrent otitis media)
Trigeminal (most common), pain usually lasts seconds, episodic
Glossopharyngeal, typically unremarkable Etiology History Audiogram & Other Tests
Geniculate, Sphenopalatine
○ Nasal Examination → check for URTI *Type B (flat): impacted cerumen, TM perforation, OME
○ HL: progressive, bilateral, symmetric
Oral Aphthous Ulcer localized pain in mouth, may refer to ear shallow mouth ulcers, usually gray ○ Oral & Pharyngeal Examination NIHL
○ Hx chronic exposure to loud noises (85 dB)
symmetric, 4kHz notch *Type C (negative pressure): Eustachian tube dysfunction
Pharyngitis,
sore throat pharyngeal erythema, tonsillar exudates
□ check teeth, palpate & percuss gums (esp molars) → dental cause of otalgia ○ HL: sudden asymmetric/unilateral, otalgia, □ Otoacoustic Emissions (OAE): for healthy newborns; evaluate cochlear (pre-neural) function only
Tonsillitis Acoustic
□ check for oropharyngeal disorders with pain that may radiate to the ear Trauma
tinnitus asymmetric, 4kHz notch □ Auditory Brainstem Response (ABR): for high risk newborns; evaluate both cochlear & retrocochlear
Salivary Gland *tonsillar exudates, pharyngeal erythema→ pharyngitis, tonsillitis ○ Hx: sudden loud noise (130 dB)
preauricular pain prominent parotid glands
disorders ○ HL: progressive (weeks), bilateral, symmetric
Sinusitis recent URTI, unusually will have otalgia nasal congestion, purulent rhinorrhea, anosmia
○ TMJ Examination ● Head & Neck Examination
○ Hx ototoxic medications (aminoglycosides, bilateral high frequencies affected first
Thyroiditis (±) painful thyroid, (±) referred otalgia (CN X) enlarged, tender thyroid □ palpation of TMJ: discomfort, crepitus → TMJ Syndrome Ototoxicity
loop diuretics, salicylates (ASA, NSAIDs), progressing to low frequencies ○ look for masses, lymphadenitis → check for cancers, infections
TMJ Syndrome pain/clicking with opening jaw Tender TMJ; crepitus on motion of mandible ○ Thyroid Exam quinine, cisplatin
□ enlarged, tender thyroid → thyroiditis with referred otalgia ○ HL: progressive, bilateral, symmetric
bilateral high frequencies affected first ● Neurologic Examination
Presbycusis progressing to low frequencies *goal: rule out neurologic causes of HL such as acoustic neuroma
SECONDARY OTALGIA: ACUTE CAUSES REQUIRING IMMEDIATE IDENTIFICATION ○ Hx older age, family history
*poor speech comprehension
Etiology History Physical Examination ● Chest Examination decrease in hearing of ≥30 dB
*in setting of COM, rule out possible meningitis especially if patient is febrile
risk factors for MI: hypertension, hyperlipidemia ○ rule out MI as cause SSNHL ○ HL: sudden (over 3 days), usually unilateral
affecting at least 3 different frequencies ○ Cerebral Exam
Myocardial Infarct CC: heavy substernal chest pain >30mins, Unstable VS □ check GSC Score
SNHL WITH VESTIBULAR SYMPTOMS
radiating to the jaw, medial arm
Older men, RF for CAD ○ HL: develops over hours to days
Audiometry: SNHL ○ Cranial Nerve Examination
Thoracic Aneurysm Unstable VS Labyrinthitis HRCT temporal bone: labyrinthine fistula □ CN VIII → Acoustic Neuroma
CC: tearing chest pain, thoracic back pain ○ PMHx: middle ear infection, URTI, meningitis
CSF Analysis: meningitis
± tenderness along temporal artery □ CN VII facial nerve palsy → Cholesteatoma
Temporal Arteritis ≥50yo, jaw claudication, diplopia ○ HL: fluctuating hearing loss
± prominent artery Meniere’s Disease
○ rotatory vertigo, tinnitus
affects low frequency first □ CN V, VII → Stroke
Acoustic Neuroma ○ HL: slowly progressive unilateral hearing loss any unilateral abnormal configuration ○ Meningeal Signs
□ (+) Brudzinski, Kernig’s, Nuchal Rigidity

LACABA 2|PAGE
UST MEDICINE – OTORHINOLARYNGOLOGY ORAL REVALIDA ΣΒΤΦ TITANS
Approach to OTOLOGIC CHIEF COMPLAINTS AUDIOLOGIC ANCILLARIES
CC: DIZZINESS ■ CC, HPI, ROS AUDIOLOGIC EXAMINATION ● Speech Audiometry
■ Common Differential Diagnosis for REVALIDA (for exhaustive differentials, refer to AAFP 2017) ■ Clinical Hearing Testing ○ affected more in retrochochlear or CNS pathology (central auditory pathways)
● Dizziness ○ measures recognition of speech rather than threshold for detection of speech signals
● Episodic Dizziness ○ description ● Finger Rub Test ○ indicate % of syllables, words, or sentences that subject has heard correctly in each test series
□ Episodic Triggered □ sensation of self-motion → vertigo ○ rub 2 fingers together 6 inches away
○ Benign Paroxysmal Positional Vertigo □ feeling of being unstable → dysequilibrium ○ (+) of failure to identify ≥3/6 attempts
○ Orthostatic Hypotension □ impending loss of consciousness → presyncope
□ Episodic Spontaneous □ non-specific light-headedness → psychiatric, metabolic ● Whispered Voice Test
○ Meniere’s Disease ○ timing ○ occlude non-tested ear; instruct the patient to repeat letter/number combination heard
○ Vestibular Migraine □ onset, duration, evolution ○ whisper letter/number combinations at arm’s length away
○ Psychiatric Conditions □ episodic vs continuous ○ (+) of failure to identify ≥3/6 attempts
○ Otosclerosis *if episodic, ask if triggered or spontaneous
*if continuous, check previous trauma or medication intake ● Speech Test
● Continuous Dizziness ○ triggers: actions, movements, situations ○ have patient repeat 2-digit numbers from 6 meters → 1 meter
○ Barotrauma, Vestibulotoxicity □ sudden changes in position → BPPV ○ at 1 meter, use normal voice, raised voice, and shout
○ Vestibular Neuritis □ movement to upright position → Orthostatic hypotension
○ Vertebrobasilar Stroke, TIA WHO Classification
● Other Questions & Associated Symptoms
Degree of Hearing Loss dB Range Performance
*the general approach to dizziness is to TiTrATE: Timing of symptoms, Triggers that provoke the symptom, and ○ fluctuating hearing loss → Meniere’s Disease ■ Tympanometry
○ Hx migraine headache → Vestibular Migraine Normal ≤ 25 no problem ○ selectively raising or lowering air pressure in the external auditory canal increases acoustic impedance
A Targeted Examination Mild 26 – 40 Hears/repeats words in normal voice at 1 meter
○ psychiatric symptoms ○ provide graphic representation of compliance changes as applied pressure is varied
○ previous history of trauma? Moderate 41 – 60 Hears/repeats words in raised voice at 1 meter
○ previous URTI → labyrinthitis Severe 61 – 80 Hears words shouted into better ear
Profound ≥ 80 Cannot hear/understand shouted voice ● Type A (Normal)
● Medication History → medication-induced ○ air pressure in ear canal & middle ear are equal
○ cardiovascular: alcohol, antiepileptics, antihypertensives ■ Tuning Fork Tests ○ tympanic membrane has lowest impedance, absorbs sound best
○ cerebellar toxicity: antiepileptics, benzodiazepine
○ ototoxicity: aminoglycosides ● Weber Test ● Abnormal
○ bleeding complications: anticoagulants *placed in midline of skull, usually vertex on forehead ○ negative pressure in ear canal (C)
○ sensorineural HL: lateralize to better hearing ear ○ lower sound absorption or compliance (B)
■ Physical Examination ○ conductive HL: lateralize to affected ear (less ambient sounds) ○ greater acoustic stiffness (As)
○ lesser acoustic stiffness (Ad)
● Vital Signs ● Rinne Test
○ blood pressure: orthostasis, hypotension *compares level of air and bone conduction in the same ear
○ check for fever if suspecting labyrinthitis *positive test: AC>BC TYPE APPEARANCE CONDITION
○ sensorineural HL: AC > BC A bell-shape (0.3cc-1.6cc) normal
● Head & Neck Examination As low compliance/height (<0.3cc) stiff ossicles
○ cochlear HL: BC > AC
Ad high compliance/height (>1.6cc) disarticulated ossicles
○ tests for hearing loss – if suspecting Meniere’s Disease
B flat tympanogram impacted cerumen, TM perforation, OME
■ Audiometry C negative middle ear pressure eustachian tube dysfunction
● Targeted Examinations
○ Dix-Hallpike Maneuver → BPPV ● Pure Tone Audiometry
○ Head Impulse, Nystagmus, Test of Skew (HINTS) Examination → Vestibular Neuritis vs VB Ischemia *approach to reading pure tone audiometry
□ Head Impulse: sitting patient, thrust head 10° to R then L while eyes are fixed on examiner’s nose (1) Is this the left or the right ear?
□ Nystagmus: patient should follow examiner’s finger as it moves slowly to left or right □ Right ○Δ<[
□ Test of Skew: ask patient to look straight ahead, cover and uncover each eye □ Left x□>]
(2) What is the degree of hearing loss?
HINTS Peripheral Central □ base this from WHO Classification (see above)
Head Impulse (+) saccade (-) saccade □ mild, moderate, severe, profound?
Spontaneous unidirectional horizontal
● Episodic Dizziness Nystagmus
nystagmus when gazing in direction of
Fast-phase alternating (changes direction with (3) What is the type of hearing loss present?
(Gaze Test)
nystagmus
gaze; horizontal or vertical) □ assess if AC is normal/abnormal ○Δx□ ■ Objective Hearing Tests for Newborns
EPISODIC DIZZINESS Vertical deviation of the covered eye after □ assess if BC is normal/abnormal <[>]
Test of Skew
Etiology History Physical Examination (Alternate Cover Test)
normal uncovering (relaxation or skew deviation after □ is there an air-bone gap? ● Auditory Brainstem Response (ABR)
○ Hx: head trauma, inner ear disorders (stapes uncovering) ○ auditory evoked potentials in the brainstem for diagnostic purposes
injury, labyrinthitis) ○ Indication: differentiate between cochlear & retrocochlear HL
BPPV
○ severe, recurrent rotatory vertigo
○ (+) Dix-Hallpike ● Cardiac Examination ○ objective measurement of hearing threshold
□ provoked by certain movements
□ fatigable
○ nausea ● Neurologic Examination ● Otoacoustic Emissions (OAE)
○ family history ○ Gait: check for unsteady gait (assess for peripheral neuropathy) ○ acoustic stimulus transmitted in retrograde fashion across ossicles to tympanic membrane
○ triad: fluctuating hearing loss, rotatory vertigo, ○ Cerebellar: Romberg’s test (dorsal column-lemniscal pathway) ○ TM: acts as membrane of loudspeaker, emitting vibrations as soundwaves into external ear canal
Meniere’s Disease tinnitus ○ Audiometry: SNHL on lower frequencies
○ nausea, vomiting, dizziness before/after the
○ Indication: test function of the cochlear amplifier (reflect integrity of the cochlea)
attack
Vestibular Migraine ○ Hx migraine + episodic vertigo - Automated Auditory Brainstem
ResponseAutomated
(AABR) tests the entire
Auditory
hearingBrainstem
pathway from the outer
Response ear
(AABR) Otoacoustic emission (OAE)
● Continuous Dizziness to the brainstem
tests theand used
entire for both
hearing tests a portion of the hearing
healthy and from
pathway high-risk newborns
the outer ear to pathway from the outer ear to
EPISODIC DIZZINESS the brainstem the inner ear/cochlea (outer hair
Etiology History Physical Examination cells)
Vestibular Neuritis ○ severe rotatory vertigo, nausea, oscillopsia
Vertebrobasilar ○ brainstem Sx: diplopia, dysarthria, weakness,
Ischemia clumsiness of the limbs
Otoacoustic Emission (OAE) tests
portion of the hearing pathway from outer
AABR technology ear to the cochlea (outer hair cells) and
OAE technology used for healthy newborns

LACABA 3|PAGE
UST MEDICINE – OTORHINOLARYNGOLOGY ORAL REVALIDA ΣΒΤΦ TITANS
SUMMARY: Approach to HEARING LOSS, OTALGIA, OTORRHEA

■ Hearing Loss ■ Otalgia ■ Otorrhea – think TM perforation ■ Otologic Physical Examination


○ laterality ○ laterality ○ laterality ○ vital signs (temperature)
○ onset/duration ○ OLDCARTS of pain ○ onset/duration ○ external ear examination
○ ear fullness ○ hearing loss/tinnitus ○ consistency (serous, bloody, purulent) ○ otoscopy
○ tinnitus ○ ear fullness ○ otalgia □ ear canal
○ dizziness – check Meniere’s disease, Vestibular neuritis ○ ear discharge – think TM perforation esp if mucopurulent ○ hearing loss/tinnitus □ tympanic membrane, mobility
○ history of trauma ○ nasal congestion ○ history of swimming ○ hearing test
○ ear manipulation – otitis externa, impacted cerumen ○ SSx of URTI – AOM □ gross hearing test/whisper test
○ history of exposure to loud noises – noise-induced hearing noise ○ history of trauma/manipulation □ tuning fork tests: weber, rinne
○ history of swimming – diffuse otitis externa ○ history of swimming – diffuse otitis externa ○ nose and throat examination
□ check for URTI

CONDUCTIVE HEARING LOSS: EXTERNAL EAR & MIDDLE EAR

DISEASE & PATHOPHYSIOLOGY EPIDEMIOLOGY ETIOLOGY CLINICAL MANIFESTATIONS DIAGNOSIS TREATMENT COMPLICATIONS, PROGNOSIS
○ pain on manipulation of pinna or tragus
OTITIS EXTERNA ○ edematous ear canal
○ infection of the skin of the external auditory canal ○ (+) infection and debris

*local mechanical trauma *otalgia and fullness ● Inspection, Palpation ○ clean with 70% alcohol (disinfection, reduce local welling)
CIRCUMSCRIBED OTITIS EXTERNA ○ Staphylococcus aureus ○ otalgia: very painful, tender swelling ○ tragal tenderness + circumscribed, very painful swelling
○ acute bacterial infection leading to furuncle ○ mild hearing loss ● Furunculosis
○ in cartilaginous portion of the ear canal ○ rarely otorrhea, afebrile ● Otoscopy ○ ototopical antibiotics, NSAIDs/analgesics, heat
○ pronounced swelling of ear canal, residual lumen
● Abscess
● Simple Hearing Test ○ incision & drainage
○ conductive HL (ear canal may be swollen shut)

● Bacteriological Exam
○ get sample from purulent center

*first and most important step:


*warm, moist climate *otalgia, crusting, purulent ear discharge ● Otoscopy: Eczema in absence of Acute Infection meticulous, repeated cleansing, and drying of ear canal ○ prevent ear manipulation
DIFFUSE OTITIS EXTERNA ○ Pseudomonas aeruginosa ○ itching – main initial symptom ○ canal skin is dry, cracked, and scaly ○ protect ears from getting wet
○ swimmer’s otitis ○ Proteus mirabilis ○ otalgia – present with acute infection ○ thickened and shows sites of desquamation ○ antiseptic antibiotic drops (antipseudomonal)
○ inflammation of the EAC skin (eczema, dermatitis) ○ crusting, purulent aural discharge (MAY occur) □ QUINOLONE drops: 3 drops 3 times a day
○ conductive HL (obstruction of ear canal) ● Otoscopy: Eczema with Infection ○ canal stenosis
○ diffuse swelling of canal skin, discharge or crusting □ steroid- and antibiotic-containing ear drops
○ fetid discharge: anaerobes!!! □ to reduce swelling, use not more than 2 weeks

● Bacteriologic Exam
○ necessary only with persistent or recurrent infection

*ear canal is locally debrided and cleaned at regular intervals


○ older patients with DM ○ Pseudomonas aeruginosa *severe otalgia, skin ulceration, canal osteitis ● Inspection: signs of infection in surrounding tissues
NECROTIZING OTITIS EXTERNA ○ initial history: persistent otitis externa, doesn’t heal ● Antibiotics
○ aka malignant otitis media ○ otalgia: moderate pain then may become severe ● Otoscopy: ○ IV antipseudomonal antibiotics or high dose antibiotics
○ caused by pseudomonas: ulceration & osteitis ○ ± fetid aural discharge ○ ulcer on canal floor, with exposed brownish bone, ○ for 6 weeks
□ on floor of the ear canal ○ fetid discharge
□ bone infection may spread to the: ● Petrostectomy
*middle ear ● Smear (with sensitivity testing!): ○ if poor response to conservative therapy or complications
*skull base ○ (+) Pseudomonas aeruginosa
*retromandibular fossa
*parotid compartment ● Radionuclide bone scan, CT:
○ define extend of infection & bone destruction

*aural toilette: it is essential that ear canal be thoroughly


*warm, moist climate ○ hearing loss ● Otoscopy cleaned and dried
OTOMYCOSIS ○ Aspergillus niger ○ otalgia, otorrhea (not much) ○ white, yellow, or black membrane
○ cerumen harbors saprophytic fungi ○ Candida albicans ○ severe itching, feeling of fullness lining the swollen, erythematous skin of ear canal ○ ototopical antimycotics
○ warm moist climate is conducive to fungal infections ○ Mucor ○ alkaline pH ○ Aspergillus: □ Nystatin solution
○ most exclusively involve bony portion of ear canal ○ dermatophytes ○ Candida: white curd-like material, can be scraped off □ Clotrimazole 3 drops 3x/day for 3 weeks
○ acidify the canal – Acetic Acid 1% 5-10 drops
● Direct Sampling: (+) mycelia ○ systemic antimycotic
□ only in IMMUNE SUPPRESSED patients

LACABA 4|PAGE
UST MEDICINE – OTORHINOLARYNGOLOGY ORAL REVALIDA ΣΒΤΦ TITANS
DISEASE & PATHOPHYSIOLOGY EPIDEMIOLOGY ETIOLOGY CLINICAL MANIFESTATIONS DIAGNOSIS TREATMENT COMPLICATIONS, PROGNOSIS
*infants (nonspecific Sx): irritability, vomiting, *± spontaneous improvement in 1-2 days without antibiotics
○ more common in children ● Bacteria (2/3 of cases) diarrhea ● Physical Examination *wait and see prescription:
ACUTE OTITIS MEDIA (eustachian tube is more ○ S. pneumoniae ○ mastoid – no swelling, moderately tender to pressure □ prescribe now but wait for 3 days before taking antibiotics
○ inflammation of middle ear <3 weeks horizontal) ○ H. influenzae ○ preceded by viral URTI □ then return after 3-5 days to check
○ usually ascending infection to middle ear via eustachian ○ Branhamella catarrhalis ○ initial Sx: severe earache ● Otoscopy
○ adenoids: frequent nidus for middle ear infections in child □ babies: rubbing the infected ear ○ opaque, thickened, erythematous, undifferentiated TM ● NSAIDs/Acetaminophen – for pain relief
● Viral (1/3 of cases) ○ fever (usually present during first 24 hours) ○ immobile by pneumatic otoscopy
○ respiratory viruses ○ (±) perforation of TM: aural discharge □ don’t do Valsalva Maneuver! ● Decongestant nose drops/irrigations: ACUTE OTITIS MEDIA
○ otalgia ○ may be necessary to relieve nasal airway obstruction
● Hearing Tests: Conductive HL ○ unobstructed nasal breathing MILD DEGREE
• Mild otalgia (VAS)
MODERATE to SEVERE
• Moderate to severe otalgia

DIFFERENTIAL DIAGNOSIS ○ Rinne’s Test (BC>AC) → improves drainage function of eustachian tube • Duration of < 48 hours
• Temperature of < 39°C
• Duration of > 48 hours
• Temperature of > 39°C

○ Hx swimming, ear manipulation ○ Weber’s Test (lateralize to affected)


Otitis Externa ○ Audiogram: abnormal AC, normal BC ● Antibiotic therapy • Analgesic for pain • Analgesic for pain
○ (+) tragal tenderness; swollen, hyperemic EAC • Observation as option • Antibiotic: oral Amoxicillin

COM in Acute ○ onset >3 months ago ○ done right away; if effective, do for 7-10 days • ENT referral for myringotomy

Exacerbation ○ TM most likely perforation, (+) secretions ● Bacteriologic Examination ○ if no response within 48 hours, use different antibiotic PREVENTIVE MEASURES

Acute OME ○ also <3 weeks, aural fullness, sloshing/popping ○ always done with spontaneous perforation; ○ if still no response, do paracentesis • Immunization (pneumococcal & HIB vaccines) and breastfeeding
• Reduce RISK factors: smoke exposure; congested living condition; daycare
○ not performed when membrane is intact □ obtain fluid for bacteriologic exam center attendance; bottle feeding/use of pacifiers; nasal allergies; craniofacial
abnormalities (cleft palate); adenoid hypertrophy

*chronic otorrhea *diagnosis is made from history and otoscopic findings


○ Pseudomonal aeruginosa *(±) hearing loss, otalgia *in acute infection & pronounced inflammation, hard to ● Acute Suppurative Episodes ● Infectious Complications – rare & atypical
CHRONIC SUPPURATIVE OTITIS MEDIA distinguish form COM + cholesteatoma even with imaging ○ repeated, meticulous cleansing & drying of ear ○ mastoiditis
○ inflammation of middle ear ≥3 months ● Acute Infection ○ adequate ear protection while bathing or showering ○ abscess formation
○ should be diagnosed in presence of: *recurrence of infection may cause ±pain ● Otoscopic Findings: Dry Ear ○ antispeudomonal antibiotic drops
□ chronic tympanic membrane perforation ○ chronic otorrhea – creamy or mucopurulent ○ central perforation of TM - NOT involve cartilaginous ring □ Ciprofloxacin/ofloxacin otic drops 3 drops 2/day x7d ● Toxic Serious Labyrinthitis
□ may be odorless & stringy ○ other features of chronic inflammation □ if ototoxic ear drops are used, only use for <3 days ○ may occur and cause cochlear hearing loss
■ Etiopathogenesis □ may be fetid if d/t Pseudomonas or anaerobes □ calcifications, atrophic areas, retractions ○ oral analgesics for pain (Paracetamol) □ on top of conductive HL
○ chronic inflammation 2° eustachian tube dysfunction ○ non-intact tympanic membrane □ ossicular destruction
○ genetic & constitutional factors ● Dry Ears: Inactive COM
□ affecting healing capacity & resistance of mucosa ● After Clearance of Infection (Inactive COM) ● Otoscopic Findings: Draining Ear ○ adequate ear protection to prevent reinfection
○ special anatomic characteristics of middle ear ○ no symptoms after infection clears ○ secretions ○ if dry for 3 months → tympanoplasty (close the TM)
□ such as pneumatization & relative sizes □ other than variable degree of hearing loss ○ may be inflamed and swollen
○ infecting organisms ○ perforation is difficult see d/t drainage & inflammation ● Chronic Intractable Suppuration
□ nature, pathogenicity, virulence, resistance ○ Valsalva maneuver may cause bubbles to appear ○ requires ablative surgery of the middle ear
□ mastoidectomy + modified radical operation
● Bacteriologic Examination: Smear
○ should be taken for DRAINING ear EAR DISCHARGE (Otorrhea)

Medical History & PE (otoscopy)


● Hearing Tests: Conductive HL
Otitis Externa Chronic Otitis Media CSF Otorrhea
*more prominent in the draining ear!
○ Rinne’s Test (BC>AC), Weber’s (lateralize to affected) Aural toilette
Topical antibiotics ABSENT PRESENT
Diagnostics: Halo sign
Beta-2-transferrin;
○ Pure Tone Audiogram: abnormal AC, normal BC Cholesteatoma Cholesteatoma Imaging: CT or MRI

Monitor & decrease ICP:


MEDICAL: WORK UP: Hearing elevate head, prevent
● Imaging INACTIVE COM aural toilette, topical
antibiotics
test, CT scan straining, give laxatives,
prophylactic antibiotics
○ not indicated in acute stage If NO response,

○ CT scan if suspecting cholesteatoma, preop, or (+) TYMPANOPLASTY SURGICAL


EXPLORATION
MASTOIDECTOMY If persistent, CSF REPAIR

complications
*presents with chronic otitis media
○ may occur in any age ● Acquired Cholesteatoma ● Otoscopic Findings ● Mastoidectomy ● Rare Complications
COM WITH CHOLESTEATOMA ○ impaired middle ear ventilation
● Dry, Uninfected Cholesteatoma ○ (+) epithelial debris in retraction pocket at the: ○ first line goal: eradicate destructive inflammatory process ○ labyrinthitis
→ negative pressure in middle ear
→ retraction pocket forms in TM ○ does NOT cause otalgia, otorrhea □ attic or posterosuperior quadrant of TM ○ surgery is necessary d/t bone destruction 2° cholesteatoma ○ facial nerve palsy
● Cholesteatoma → squamous epithelium migrate ○ aural pressure (impaired middle ear ventilation) ○ (±) bony erosion in the posterosuperior canal close to TM ○ intracranial infection (meningitis, brain abscess)
○ keratinizing squamous epithelium found in bony spaces To the external canal ○ conductive hearing loss ○ if (+) dry cholesteatoma ● Tympanoplasty
○ bone is destroyed via inflammatory osteoclastic process ○ (±) functional deficits (eventually) □ brownish-black crusts in superior canal wall ○ second line goal: improve hearing
□ facial nerve palsy, vestibular dysfunction ○ if (+) acute inflammatory changes
□ difficult to accurately Dx COM + cholesteatoma ● Acute Inflammatory Changes
● Infected Cholesteatoma with Discharge ○ treat like chronic suppurative otitis media
○ COM with fetid otorrhea ● Workups ○ give the patient the medications, immediately work him up
○ conductive hearing loss ○ hearing tests for surgery, and do mastoidectomy as soon as possible!
○ (±) functional deficits (eventually) ○ CT scan *you can do mastoidectomy even if with suppuration!
□ facial nerve palsy, vestibular dysfunction

○ no organism cultured *does NOT present with acute otologic Sx ● Otoscopy (OME is diagnosed otoscopically!) ● ACUTE/SUBACUTE: Conservative Treatment
OTITIS MEDIA WITH EFFUSION *does NOT present with systemic signs ○ opaque tympanic membrane, poor mobility *done especially in children with acute or subacute OME
○ inflammatory effusion behind an intact TM ○ (+) air bubbles if PARTIAL, not in complete ○ relieve nasal airway obstruction
○ there is no infection, just inflammation ○ history of colds (often precedes complaints) ○ Valsalva: does not force air into tympanic cavity ○ treat infection in nose and paranasal sinuses
○ clogged pressure or sensation in affected ear ○ perform regular Valsalva Maneuvers
■ Partial vs Complete Middle Ear Effusion ○ pain is RARELY present ● Hearing Tests: Conductive HL
○ partial OME: subacute, serous inflammation ○ hearing loss ○ Rinne’s Test (-) ● CHRONIC: Surgical
○ complete OME: mostly chronic ○ popping/sloshing sound ○ Weber’s Test (lateralize to affected side) *persists for > 3 months with TM becoming thickened
○ Tympanogram: Type B (flat curve) ○ Myringotomy with ventilation tube insertion
■ Classification According to Timing ○ Audiometry: abnormal AC, normal BC; air-bone gap
○ acute OME ≤ 3 weeks ○ OAE: absent
○ subacute OME ≤ 3 months
○ chronic OME > 3 months *if persists for >3 months, r/o tumor by Endoscopic Exam
of Nose & Nasopharynx

LACABA 5|PAGE
UST MEDICINE – OTORHINOLARYNGOLOGY ORAL REVALIDA ΣΒΤΦ TITANS
SENSORINEURAL HEARING LOSS

DISEASE & PATHOPHYSIOLOGY EPIDEMIOLOGY ETIOLOGY CLINICAL MANIFESTATIONS DIAGNOSIS TREATMENT COMPLICATIONS, PROGNOSIS
*there is no known effective treatment
● Acute Acoustic Trauma ● Acute NIHL ● Pure Tone Audiometry *prevention is most important for NIHL ● Decompensated Tinnitus
NOISE-INDUCED HEARING LOSS ○ >140dB for <1.5ms ○ muffled hearing ○ threshold shift: drop off threshold bet 3-6 kHz ○ results in significant psychological stress
○ hearing loss due to excessive noise exposure causing ○ tinnitus □ 4 kHz notch – hallmark! ■ Prevention
direct mechanical trauma to the delicate structures of the ● Blast Injury *improves after noise is withdrawn □ acute noise exposure: temporary threshold shift
cochlea ○ >140dB for >2ms □ chronic noise exposure: permanent threshold shift ● Primary Prophylaxis
● Irreversible Chronic NIHL *over time: irreversible, progressive high-tone HL ○ reduction of noise emissions
■ Types of Noise-Induced Hearing Loss ● Chronic NIHL *progressive, bilateral, symmetric SNHL *may affect middle frequencies 1-2 kHz □ <85 dB for <8 hours per day is safe
*distinguished based on time exposure, exist as a continuum
*severity based on noise level, exposure time, individual factors ○ >85dB for >8hrs ○ features of SNHL ○ hearing protection
□ loss of speech discrimination
● Acute Acoustic Trauma □ difficulty hearing with background noise ● Secondary Prophylaxis
○ exposure to sudden intense sound event of shrot duration ○ constant tinnitus *for those with occupational noise exposure
○ >140dB for <1.5 milliseconds ○ regular screening
● Blast Injury ○ hearing protection, change of workplace
○ pressure wave from an explosive blast
○ >140dB for >2 milliseconds

● Acute Noise-Induced Hearing Loss


*often reversible or partially reversible
○ exposure to high levels of continuous or intermittent (pulsed or impact)
noise lasting from seconds to hours

● Chronic Noise-Induced Hearing Loss


*irreversible cochlear hearing loss
○ >85dB for >8 hours per workday

*progressive, bilateral, symmetric SNHL *no specific medical or surgical treatment


>50 years old ○ old age *initial signs appear as early as 4th decade ● Otoscopy
PRESBYCUSIS ○ normal ○ bilateral hearing-aid fitting
○ age-related, apparently idiopathic & symmetrical SNHL ○ hearing loss ○ rehabilitative measures to improve communication
○ difficulty recognizing speech in background noise ● Hearing Tests: SNHL
■ Pathogenesis ○ ± tinnitus ○ Tuning Fork Tests
*collective, encompass various disorders of auditory system □ Rinne’s Test (AC>BC)
and various etiologic influences □ Weber Test (lateralize to normal ear)
○ aging: autointoxication, loss of elasticity ○ Audiometry
○ genetic predisposition □ Impedance: normal
○ cumulative exposure that damage hearing □ PTA: symmetrical SNHL (abnormal AC, BC, no gap)
□ noise, middle ear diseases, toxins, tobacco use □ Speech: impaired speech recognition than expected

*speech audiometry: evaluate understanding/comprehension


(retrocochlear/CNS) rather than pure tone (cochlear)

*progressive, bilateral, symmetric SNHL


*ototoxic effects are usually asymmetrical ○ otoscopy: normal ● Primary Prophylaxis – if use is planned
OTOTOXICITY ○ tuning fork tests: SNHL ○ pretreatment PTA, high-tone audiometry, OAE
○ toxic damage to the inner ear ● History ○ PTA: symmetrical SNHL (abnormal AC, BC, no gap) ○ regular measurement of ff during treatment:
○ may affect both cochlear & vestibular functions ○ cochlear disorder □ serum drug levels
○ may be reversible or irreversible □ tinnitus – may be initial symptom □ dose, renal function, adequate hydration
□ symmetrical hearing loss
○ vestibular disorder ● Secondary Prophylaxis – early detection
□ NO labyrinthine imbalance, NO nystagmus ○ PTA, high-tone audiometry, OAE upon CC: tinnitus,
□ oscillopsia, dysequilibrium vestibular problems, subjective hearing difficulties

● Medication History
○ aminoglycosides, loop diuretics
○ salicylates, quinine, cisplatin

*sudden (3 days), unilateral or asymmetric SNHL *symptomatic treatment is treated as soon as possible
○ viral ○ otoscopy: normal ■ Prognosis
SUDDEN SENSORINEURAL HEARING ○ vascular ○ sudden SNHL occurring over 3 day period ○ tuning fork tests: SNHL ● Causal Treatment ○ >50% experience spontaneous resolution in the first few
LOSS ○ autoimmune ○ tinnitus (often) ○ PTA: unilateral, decrease in hearing of ≥30dB, ○ initiate as soon as possible weeks regardless of treatment
○immediate unilateral hearing loss with no apparent external ○ vestibular symptoms (less common) affecting at least 3 consecutive frequencies ○ treat: labyrinthitis, herpes zoster oticus, perilymphatic fistula
cause
● Idiopathic Sudden Hearing Loss
○ aim: improve microcirculation & oxygenation of inner ear
○ initial treatment: oral corticosteroids 5 days (short course)

LACABA 6|PAGE
UST MEDICINE – OTORHINOLARYNGOLOGY ORAL REVALIDA ΣΒΤΦ TITANS
DIZZINESS

DISEASE & PATHOPHYSIOLOGY EPIDEMIOLOGY ETIOLOGY CLINICAL MANIFESTATIONS DIAGNOSIS TREATMENT COMPLICATIONS, PROGNOSIS
*before starting, confirm patient details, ask for
○ adults > children ○ head trauma ● History contraindications (eg neck surgery/trauma), tell the patient ● Epley Maneuver
BENIGN PAROXYSMAL POSITIONAL ○ women > men (2:1) ○ prior Hx vestibular neuronitis ○ severe, recurrent rotatory vertigo ~1 min the maneuver and ask patient to please maintain open eyes *designed to displace and reposition the canalith
VERTIGO ○ most common! ○ inner ear disorders (eg. previous □ provoked by certain movements, fatigable (1) patient sitting up, turn head 45° to one side
○ a peripheral disorder stapes injury or labyrinthitis) □ more intense following rest ● Dix-Hallpike Maneuver (2) lie patient down, head hanging 20-30° at edge of bed
○ due to canalolithiasis □ may be awakened at night d/t vetigo *start with the side of patient’s complaint □ observe for nystagmus, maintain position for 30-60s
□ particles (usually dislodged otoconia) ○ nausea (1) with patient sitting up, turn head 45° to one side (3) turn patient’s head 90° to opposite side (45° to other side)
□ floats around in endolymph of semicircular canal ○ no associated symptoms (2) lie patient down with head overhanging edge of bed □ maintain position for 30s
○ otoconia cause unphysiologic deflection of cupula □ neck should be hanging 20-30° (4) ask patient to turn his body to the side where head is turned
□ look for rotatory nystagmus to one side after 10s □ maintain head turn such that Px now looks towards floor
□ crescendo-decrescendo □ maintain position for 30-60s
*increase for 30s, then diminishes (5) ask patient to sit up sideways, maintain head rotation
(3) brough back to upright (6) once sitting upright,
□ similar nystagmus occurs in opposite direction □ head can be midline, neck can be flexed
(4) do the same for the other side □ maintain position for 30s

*Results of Dix-Hallpike ● Surgery – RARELY DONE


○ negative but suspected: repeat on the other side ○ Singular Neurotomy: surgical disruption of neural connection
○ up beat nystagmus: posterior BPPV to ampulla of posterior semicircular canal
○ lateral beat nystagmus: lateral BPPV ○ blockage of posterior semicircular canal
○ down beat nystagmus: anterior BPPV upright 24 hours then collar

with companion, bawal pa magdrive,


*poorly understood, mostly ● Classical Clinical Triad ● Typical History ● Acute Attack
MENIERE’S DISEASE idiopathic ○ attacks of rotatory vertigo lasting several hours ○ vertigo, tinnitus, fluctuating hearing loss ○ bed rest
○ aka endolymphatic hydrops ○ tinnitus (continuous, but varying intensity and low ○ anti-vertiginous or antiemetic medications (thiethylperazine,
○ generally unilateral inner ear condition characterized by ○ some (+) FHx intensity “roaring”) ● Otoscopic Findings dimenhydrinate, metoclopramide)
the classic clinical triad of: ○ rarely from previous inner ear ○ fluctuating hearing loss (and dysacusis) ○ normal (idiopathic forms) ○ fluid & electrolyte replacement: if with profuse vomiting
□ attacks of vertigo lasting several hours damage (eg post-traumatic cochlear ○ late signs of previous inflammation (symptomatic)
□ tinnitus hydrops) ● Other Symptoms ● Prevention of Attacks: Medical Therapy
□ fluctuating hearing loss ○ nausea and vomiting ● Audiometry *done in between attacks, initially for frequent attacks
○ disequilibrium, dizziness *Pure Tone Audiometry ○ antivertiginous antihistamines (betahistine)
□ before and long after the attacks ○ sensorineural hearing loss, affecting low frequencies ○ centrally-acting CCB (cinnarizine)
○ later affects all frequencies (pantonal hearing loss)
*Speech Audiometry ● Prevention of Attacks: Surgery
○ disproportionate hearing loss for speech (mechanical *if symptoms do not improve with medical treatment → destroy
distortions of cochlea) portions of the labyrinth
○ intratympanic gentamicin (instilled into middle ear through
● ABR Testing: normal myringotomy tube)
○ vestibular neurotomy
○ labyrinthectomy (surgical ablation, sacrificing hearing; only in
patients who are deaf)

labrynitis, vestibular schwanoma, vestibular neuritis, labyrinhitis (posible


abnormal otoscopic exam, previous middle ear infection, Accute OM,
perforated tympanic membrane)

LACABA 7|PAGE
UST MEDICINE – OTORHINOLARYNGOLOGY ORAL REVALIDA ΣΒΤΦ TITANS

RHINOLOGY
Approach to RHINOLOGIC CHIEF COMPLAINTS
RHINOLOGIC CHIEF COMPLAINTS RHINOLOGIC HISTORY & PHYSICAL EXAMINATION DDx: RHINITIS
■ Common Differential Diagnosis for REVALIDA ■ History
*the usual chief complaints are nasal congestion/obstruction and rhinorrhea *nasal symptoms with triggers (rhinitis) → nasal symptoms + fever + facial pain (rhinosinusitis) ALLERGIC RHINITIS
*highlighted in blue are mentioned in initial revalida topic outline ○ immediate IgE-mediated reaction to foreign substances (allergens)
*highlighted in red are mentioned in the final revalida topic outline ● CC, HPI ○ classified according to presence of allergen
○ rhinorrhea Etiopathogenesis
□ seasonal (hay fever): allergen spectrum (eg ends after pollen season)
● CC: Rhinorrhea □ onset, duration, frequency □ perennial: year-round exposure, permanent inflammation of nasal mucosa
○ External Nose Inflammation □ laterality, character of secretions Epidemiology
□ Nasal Furunculosis *odorous, unilateral → nasal foreign body ○ obstructed nasal breathing and sneezing attacks
□ Erisypelas *watery anterior rhinorrhea → allergic rhinitis Symptoms ○ watery nasal discharge
○ Nasal Cavity Inflammation ○ nasal obstruction ○ itching of the nose and eyes (conjunctivitis).
□ Allergic Rhinitis □ onset, duration, frequency ○ detailed allergic history
□ Non-Allergic Non-Infectious Rhinitis: Occupational Irritant Rhinitis □ laterality: unilateral or bilateral? ○ allergy testing: identify allergens involved
○ Other Causes *unilateral → septal deviation, JNAF, NPCA, foreign body Diagnosis
○ seasonal allergic rhinitis: bluish purple discoloration of mucosa
□ CSF Leak *bilateral → adenoid hypertrophy, nasal polyp, rhinitis, rhinosinusitis ○ perennial rhinitis: bright red mucosa with inflammatory changes
○ hyposmia/anosmia
○ avoid contact with allergen
● CC: Nasal Congestion/Obstruction □ sudden loss → viral infections, head injury, skull base fracture, chemical/drug radiation
Treatment ○ pharmacologic treatment (especially why allergen avoidance is impossible)
○ Mucosal: Rhinitis & Rhinosinusitis □ gradual loss → deviated septum, mucosal swelling, polyps, tumors
□ mast-cell stabilizers, local and systemic H1 antihistamines, local steroids
□ Allergic Rhinitis ○ possible causes, aggravating factors
□ Non-Allergic Non-Infectious Rhinitis: Occupational Irritant Rhinitis *goal: rule out aggravating factors for all subtypes of rhinitis!!!
□ Acute Bacterial Rhinosinusitis □ relation to temperature change → vasomotor rhinitis (worse on cold) ACUTE RHINITIS DDx: NASOPHARYNGEAL MASS
□ Chronic Rhinosinusitis with Nasal Polyps □ sources of allergens at home (pollens, pets) → allergic rhinitis ● Etiology: Rhinovirus, Coronavirus (50% of causes of acute viral rhinitis)
○ Anatomic: Mechanical & Structural □ occupational exposure → occupational rhinitis ○ other pathogens: influenza virus, adenovirus Condition Adenoids Juvenile Angiofibroma Nasopharyngeal CA
Etiopathogenesis
□ Septal Deviation, Nasal Bone Fracture □ family members, close contacts with similar symptoms → infectious rhinitis ○ MOT: droplet infection (airborne) Age/Gender children (adenoids grow in adolescent males (14-25) males 50-60 years old
○ incubation: 3-7 days Predilection childhood and regress during
□ Nasal Foreign Body □ long-term decongestant use → rhinitis medicamentosa adolescence)
○ Neoplastic: Nasopharyngeal Masses ○ sleep quality, impairment of ADL/work/school → allergic rhinitis Epidemiology ○ most prevalent infectious disease (does not confer postinfection immunity) Risk Factors URTI (viral & bacterial), hormonal influence (?) genetics (southern Chinese),
□ Adenoid Hypertrophy laryngopharyngeal reflux, EBV infection, dietary
□ Juvenile Nasoangiofibroma ● ROS, Associated Symptoms ● Dry Stage (beginning of disease) allergies, exposure to cigarette (salted/smoked fish;
□ Nasopharyngeal Carcinoma ○ fever, malaise → possible infection ○ malaise (lethargy, headache, fever) smoke nitrosamines/
preservatives)
○ facial pain → sinusitis (check for sinus tenderness) ○ local discomfort in nose & nasopharynx (burning, soreness) Clinical mouth breathing, nasal obstructed nasal breathing, unilateral conductive HL,
Unilateral Nasal Obstruction ○ itching, sneezing, coughing → allergic rhinitis Manifestation discharge (purulent rhinorrhea), severe recurrent epistaxis, middle ear effusion, cervical
Anatomic Neoplastic Infectious ○ teary eyes, conjunctival injection → allergic rhinitis ● Catarrhal Stage (follows dry stage) snoring, anorexia, hyponasl headache, middle ear LN metastasis, epistaxis,
○ symptoms of AOM (otalgia, ±otorrhea, -tragal tenderness, <3 weeks) → associated with NP masses ○ watery, initially serous nasal discharge voice, OME effusion unilateral nasal obstruction,
BENIGN Symptoms headache, CN palsies
inverting papilloma, juvenile Odontogenic sinusitis ○ nasal obstruction d/t mucosal swelling (mainly turbinates)
others:
Septal Deviation ● PMHx ○ profuse nasal discharge, inflammatory changes in the vestibule
nasoangiofibroma Fungal rhinosinusitis postnasal drip, fever, halitosis,
Concha bullosa ○ history of facial trauma → nasal fracture, septal deviation nasal obstruction, adenoid
MALIGNANT Foreign body
squamous cell CA, NPCA *viruses damage mucociliary transport, hamper normal clearing of secretions facies
● Family History *viruses damage epithelium, promoting bacterial colonization (discharge become
Diagnosis midline mass in the well circumscribed, nasal endoscopy: ranges
○ allergies, atopic dermatitis → allergic rhinitis mucopurulent) nasopharynx, commonly in vascularized mass with from smooth, well-
Bilateral Nasal Obstruction
children superficial vascular markings circumscribed surface to
Anatomic Neoplastic Mucosal ● Medication History Diagnosis in the nasopharynx mucosal ulcerations
Allergic Rhinitis ○ α adrenergic decongestants >5-7 days → rhinitis medicamentosa ○ decongestant: supportive; relieve nasal obstruction & prevent sinusitis
Adenoid Hypertrophy
Infectious Rhinitis Ancillaries endoscopy, hearing evaluation MRI or CT scan, digital biopsy: for hisologic
Turbinate hypertrophy Nasal Polyps ○ nose drips: no longer absolutely necessary (<1 week) d/t risk of tachyphylaxis (audiometry) subtraction angiography confirmation
Non-allergic Non-infectious ■ Physical Examination Treatment
Nasal valve insufficiency ○ chamomile steam inhalation, “light baths,” and infrared therapy (identify tumor-feeding
rhinitis ○ antibiotics if with bacterial superinfection vessels)
● Vital Signs Endoscopic symmetric, diffuse mass origin in the lateral nasal asymmetric mass in
● CC: Hyposmia/Anosmia ○ fever → infection, acute bacterial rhinosinusitis Findings without invasion of nearby cavity near sphenopalatine Rosenmuller’s fossa;
NON-ALLERGIC NON-INFECTIOUS RHINITIS structures foramen; aggressive spread locally,
○ No memory of olfaction
HORMONAL RHINITIS Locally aggressive & intracranially, or
□ Congenital: Kallmann Syndrome (olfactory bulb aplasia) ● HEENT Examination VASOMOTOR RHINITIS ATROPHIC RHINITIS
RHINITIS MEDICAMENTOSA invades surrounding systemically (lungs, liver,
○ Sudden loss ○ Inspection: nasal crease, allergic salute, cheilitis, Dennie-morgan lines, allergic shiner → allergic rhinitis tissues & bone through bones)
□ Post-infectious: viral infection (Acute Rhinosinusitis) ○ Palpation: sinus tenderness? neurovascular autonomic unknown, iatrogenic pregnancy- side effect of long- pressure resporption
□ Post-traumatic: head injury, skull base fracture, chemical/drug radiation exposure disturbances in (botched septoplasty, associate rhinitis; term use of Treatment ○ Adenoidectomy: surgical ○ treatment of choice: ○ high-voltage
○ Eyes: teary eyes, conjunctival injection → allergic rhinitis regulating tonus of nasal excessive estrogen induced decongestant nose procedure to remove the sugical excision radiotherapy: NPCA is
○ Gradual loss ○ Ears: signs of AOM (- tragal tenderness, conductive HL) Etio
mucosal vessels chonchotomy), swelling of mucosa drops enarged adenoids radiosensitive!
□ Degenerative: Alzheimer’s Disease, Parkinson’s Disease, Diabetes Mellitus pathogenesis
○ Nose: anterior rhinoscopy: color of mucosa, turbinates, septal deviation, polyps Klebsiella ozaenae with nasal airway ○ preoperative
□ Sinunasal: deviated septum, mucosal swelling, polyps, tumors ○ Mouth: postnasal drip, tonsils Hx: related to obstruction ○ Myringotomy: paracentesis embolization of feeding ○ unfavorable tumor location
□ Idiopathic: temperature change of fluid behind tympanic vessel: reduce intraoperative precludes curative surgery
○ Neck: check for lymphadenopathies (viral infection, NPCA) membrane followed by bleeding
ventilation tube insertion for
● Pulmonary Examination obstructed nasal pronounced dryness of obstructed nasal drainage
○ auscultate the chest – wheezing? Signs of asthma? breathing, watery nasal nasal mucosa; breathing, dry
discharge, sneezing mucosa, occasional
fetid nasal odor olfactory
Symptoms
(bacterial colonization, disturbances
not perceived by Px
due to degeneration of
olfactory epithelium)

inspection: similar to broad nasal cavity lined


Diagnosis allergic rhinitis with dry crusted
mucosa

antihistamines, conservative: saline diminish as term


corticosteroid nasal nasal douche, soothing approaches &
sprays mucosal ointments disappear after the
delivery
intractable vasomotor surgical (if conservative
Treatment
nasal rhinitis: treatments are
electrocoagulation, laser inadequate):
ablation, mucotomy submucous
implantation of cartilage
grafts

LACABA 8|PAGE
UST MEDICINE – OTORHINOLARYNGOLOGY ORAL REVALIDA ΣΒΤΦ TITANS
DISEASE & PATHOPHYSIOLOGY EPIDEMIOLOGY ETIOLOGY CLINICAL MANIFESTATIONS DIAGNOSIS TREATMENT COMPLICATIONS, PROGNOSIS

ALLERGIC RHINITIS DIFFERENTIAL DIAGNOSIS ■ History *diagnosis remains clinical ● Complications


○ disorder induced after allergen exposure Non-Allergic ○ no nasal itching ● ≥2 of the ff for >1 hour on most days: *but skin testing may be needed in some cases ○ acute or chronic sinusitis
○ due to an IgE-mediated inflammation Rhinitis ○ (+) environmental irritants ○ watery anterior rhinorrhea ○ chronic allergic inflammation
○ purulent discharge ○ nasal obstruction (congestion) ● ARIA Classification of Allergic Rhinitis (2008) → adenoid & tonsillar hypertrophy
Rhinosinusitis ○ sneezing attacks ○ ARIA: allergic rhinitis impact on asthma → eustachian tube obstruction
○ no allergic symptoms
○ nasal pruritus ○ persistent if ≥4 days per week AND ≥4 weeks at a time → otitis media, obstructive sleep apnea
Vasomotor ○ related to temp change, hot
○ ± conjunctivitis ○ upper respiratory infection
Rhinitis liquid, or alcohol
□ conjunctival injection, itchiness, teary eyes ○ nasal polyps
○ mouth breathing, discolored
Adenoidal ○ hearing/speech development impairment
nasal secretions, snoring
Hypertrophy ● Associated Symptoms ○ eustachian tube dysfunction
○ no AR features ○ cough d/t postnasal drip
○ wheezing (esp. if with asthma)
○ hyposmia, anosmia

● Tertiary History ● Step 1: Allergen Avoidance


■ Classification of Allergic Rhinitis ○ tobacco use, air pollution ○ best treatment strategy
*according to presence of allergen in the environment ○ ticks, house pets, house dust mites, cockroaches ○ eliminate allergenic irritants from environment
○ house dust mites (87%), cockroach (41%), molds
● Seasonal (Hay Fever) ■ Signs
○ depends on allergen spectrum of the patient ● Detailed Allergen History ● Step 2: Pharmacotherapy
○ usually d/t pollens, disappear by end of pollen season ● General Observation ○ symptoms present year-round or only with contact (2) Nasal saline irrigation – as adjunct
○ rabbit nose ○ symptoms disappear during vacation? □ Flo sinus
● Perennial ○ allergic salute → transverse nasal crease (3) Oral antihistamines – 2nd gen > 1st gen
○ year-round allergen exposure ○ allergic cluck ● Allergy Testing (Epicutaneous skin test) □ Loratadine 10mg OD before bedtime
○ permanent causative allergens ○ allergic gape → halitosis ○ to identify the allergen involved (4) Intranasal Antihistamines – alt to oral, not adovated
○ eg causes: house dust mites, pet dander, molds, ○ allergic shiners (5) Intranasal corticosteroids
occupational, cockroaches ● Rhinoscopy □ Fluticasone propionate (Nasoflo)
● HENT Exam ○ seasonal: bluish-purple pale, boggy turbinates (6) Short course oral corticosteroids (1 week)
○ Nose: pale boggy turbinate w/ clear secretion ○ perennial: bright red, inflammatory changes □ for refractory to oral antihistamine & INCS
○ Mouth: (7) Oral LTRAs +/- other agents
□ cobblestoning of posterior pharynx *nasal endoscopy is NOT indicated unless considering (8) Intranasal Cromolyn – if refractory to ALL OTHER
□ high arch palate concurrent diseases such as nasal polyps (9) Oral & Topical Decongestants for nasal obstruction
○ (±) postnasal drip
*Patient instruction:
(1) Use NSI: blow the nose first, gently
○ lean forward, insert the nasal spray laterally, squeeze
○ wait for 10-15 minutes until dry
(2) afterwards, 2 sprays per nostril Fluticasone
(3) Loratadine 10mg OD before bedtime

● Step 3: Allergen-Specific Immunotherapy


○ do epicutaneous skin test beforehand!
○ can be done once allergen is identified

● Risk Factors
OCCUPATIONAL RHINITIS ~5-15% ○ atopic disorders: allergic rhinitis, asthma
○ high levels of IgE
■ Symptoms ● Rhinoscopy ● Avoidance ■ Prognosis
○ episodic inflammatory condition of the nose ○ known workplace respiratory irritants
○ nasal congestion ○ swollen nasal turbinates ○ eliminate or minimize exposure ○ may have stable OR over a period of years
□ intermittent/persistent symptoms ○ nonspecific nasal symptoms ○ sneezing ○ minimal nasal findings → Annoyance OR ○ substitution of causative agent ○ medication alone is NOT sufficient, reduction or
○ dust, strong odors ○ nasal pruritus ○ ulcerated, hemorrhagic mucosa → Corrosive OR ○ consistent PPE use cessation of exposure is necessary
*nasal congestion, sneezing, rhinorrhea, itching
□ and/or variable nasal airflow limitation *smoking has not been established as a risk factor for ○ and/or increased nasal secretion *exclude infectious rhinosinusitis, allergic rhinitis ○ improved ventilation or containment of causative agent ○ there is a risk for progressing to occupational asthma
OR, but increases risk of developing occupational ○ reduce exposure times to causative agent
□ and/or hypersecretion asthma
● Timing of Symptom Onset
○ due to causes & conditions attributable to particular work ● Confirmation of Work Relatedness ■ Prevention
● High Risk Occupations ○ onset of symptoms at work
○ often coexists with occupational asthma ○ furriers, livestock breeders, farmers, vets ○ improvement on weekends or vacations
○ accomplished with site visit ● Pharmacotherapy ○ information campaigns & educational tools for workers
□ exposure to animal proteins ○ recurrence of symptoms on reexposure *for allergic, irritant, and annoyance OR ○ environmental control – cornerstone of prevention
○ bakers, food processing workers ● Direct Nasal Challenge ○ intranasal glucocorticoid sprays ○ preapprentice health assessment + medical surveillance
■ Pathogenesis ○ assemblers of electrical, electronic, & telecom
○ may not appear until several hours after leaving workplace
products ○ appear within first 1-3 years of employment *aka Nasal Provocation Test □ Fluticasone proprionate 27.5mcg/spray 2 sprays OD ○ screening applicants for jobs with risk of workplace
*hyperresponsiveness of afferent sensory limb ○ pharmacy professionals
→ exaggerated efferent response to neuronal stimuli ○ GOLD STANDARD FOR DIAGNOSIS OF OR!!! *2nd generation, less bioavailability sensitizations
*office workers are low risk! ■ Occupational History
→ oversecretion of mucous, increased nasal congestion ○ current occupation, previous occupation
○ quantify response by symptom score or rhinomanometry □ Budesonide 32 mcg/dpray 1-2 sprays OD
■ Exposures (checks airway resistance) ○ and/or non-sedating antihistamines
○ exposure to cold, ozone, pollution, tobacco, animals, dusts
■ Differentiate from Work-Exacerbated Rhinitis ● Occupational Allergic Rhinitis ○ similar symptoms in coworkers ○ nasal response is gauged before, during, and after typical □ Loratadine 10mg OD
○ rodent allergens (lab workers) ○ duration of working at current job shift □ Cetirizine 10mg OD
○ pre-existing or concurrent allergic/non-allergic rhinitis ○ latex (HCW)
○ adequacy of ventilation
○ worsened but not caused by workplace exposure ○ guar gum (carpet workers) ○ adjunctive therapies
○ psyllium in laxatives (nurses) ○ compliance with PPE
○ history of accidents or spills *Methacholine Challenge Test: used to test glandular □ Iptratropoum nasal spray
● Occupational Irritant Rhinitis hyperresponsiveness, not unique to NAR □ Leukotriene-modifiers: Montelukast, ZIleutin
○ paints, talc, coal dust □ nasal irrigation with saline
○ air pollutants, cold air
● Past Medical History
○ cleaning products (janitors) ○ atopy
*immunotherapy is not recommended for majority of patient
● Occupational Corrosive Rhinitis ● Medication History
*irritating and soluble chemical gases
○ intranasal medications (OTC decongestants) with OR because allergen immunotherapy has NO ROLE in
*high water solubility & chemical reactivity management of irritative or corrosive OR
○ chlorine, sulpur oxide, ammonia
○ formaldehyde ● Current Health & Risk Factors
○ tobacco use
● Occupational Annoyance Rhinitis
*heightened olfactory awareness
○ perfumes, detergents (dept store/sueprmarket) ● Family History
○ likelihood of developing: ○ atopy
□ polyposis, sinusitis, tobacco
□ overused OTC decongestants, illicit drugs

LACABA 9|PAGE
UST MEDICINE – OTORHINOLARYNGOLOGY ORAL REVALIDA ΣΒΤΦ TITANS

DISEASE & PATHOPHYSIOLOGY EPIDEMIOLOGY ETIOLOGY CLINICAL MANIFESTATIONS DIAGNOSIS TREATMENT COMPLICATIONS, PROGNOSIS
*in BLUE are recommendations of PSOHNS 2016
● Children ● Initial Dry Stage ● Definition of Rhinosinusitis ■ Management based on Visual Analog Scale
ACUTE RHINOSINUSITIS ○ ethmoid (predominant) ○ malaise ○ ≥2 of the following for <12 weeks: ■ Visual Analog Scale & Inspection of Nasal Cavity ○ VAS <5 symptom relief (analgesic, decongestant)
○ rhinogenic type of sinusitis lasting <12 weeks ○ local discomfort □ sudden onset nasal blockage/congestion ○ VAS 5-7 intranasal corticosteroids
□ note: not all sinusitis are due to rhinitis, eg: ● Adults (MEFS) □ OR nasal discharge (nasal drip) ● Visual Analog Scale (VAS) ○ VAS 8-10 intranasal corticosteroids + Oral Amoxicillin
□ dentogenic sinusitis, *most to least common ● Catarrhal Stage □ ± facial pain or pressure ○ subjectively quantifies patient’s symptom severity
○ acute sinusitis 2° spread from intranasal inflammation ○ maxillary ○ watery nasal discharge □ ± hyposmia or anosmia ○ answer to the question: “how troublesome are tour symproms of
○ ethmoid ○ nasal obstruction rhinisinusutis?”
○ frontal *adequate hydration
○ sphenoid ○ postnasal drip
○ recurrent headaches ● Viral
○ 0.65% NaCl spray 2 sprays/nostril q6h for
1 week
○ analgesics q6h prn for facial pain
○ chamomile steam inhalation
● Rhinoscopy or Nasal Endoscopy ○ NO ANTIBIOTICS
● SSx of Acute Rhinosinusitis *rhinoscopy should always be done
○ acute rhinitis *nasal endoscopy may be used to examine nasal cavity &
● Bacterial
○ headache exacerbated by bending over nasopharynx for anatomical abnormalities & origin of purulent ● Watchful Waiting
□ pain most intense over affected sinuses discharge ○option for uncomplicated ABRS provided there is good ff-up ○ PLUS Antibiotics
○ pus tracking along middle meatus of nasal cavity □ Temperature <38°C □ Amoxicillin or Co-Amoxiclav
Affected Sinus Location of Pain ○ purulent track may NOT be seen if mucosa is swollen □ no extra-sinus complications *40-50mg/kg/day 3 divided doses x7d
Maxillary maxillary sinus + adjacent midface & temple
Ethmoid bridge of nose, medial canthus of the eye ○ sphenoid sinusitis
Frontal
anterior wall & floor of frontal sinus □ found about the ostium in anterior wall of sphenoid ■ Conservative Therapy ● Maxillary Sinus Puncture
may radiate to medial canthus
fairly nonspecific
□ or posterior wall of the pharynx ○ decongestant nose drops ○ does not respond to treatment
Sphenoid dull aching pressure at center of skull ○ nasal spray
radiating to the occiput ○ inserting cotton pack soaked with nose drops
→ into middle meatus
○ adjunct: heat therapy & inhalation of chamomile or sage
● Viral Causes ● Acute Viral Rhinosinusitis ● Complication
ACUTE VIRAL RHINOSINUSITIS ○ Rhinovirus *duration: <10 days ● First-Line Antimicrobials for 7-10 days ○
○ RS 2° virus lasting for <10 days, <12 weeks ○ nasal congestion ○ patients with low risk for antimicrobial resistance
○ usually lasting up to 4 weeks (28 days) ○ and/or nasal discharge (nasal drip) □ Amoxicillin-Clavulanate 625mg q8h or 1g BID
○ ± facial pain or pressure □ Amoxicillin 500mh q8h or 1g BID
○ ± reduction or loss of smell ○ penicillin allergy
□ Doxycycline 100mg q12h
□ Levofloxacin 500mg OD
● Bacterial Causes ● Acute Bacterial Rhinosinusitis ■ Imaging □ Moxifloxacin 400mg OD ● Complication
ACUTE BACTERIAL ○ Haemophilus influenzae *duration: ≥10 days, <12 weeks ○ NOT recommended for routine diagnosis of ABRS ○ can reach brain parenchyma
RHINOSINUSITIS ○ Streptococcus pneumoniae *persistent AVRS symptoms after 10 days plus: ○ reserved for patients with: ● Second-Line Antimicrobials
○ RS 2° bacteria lasting for ≥10 days, <12 weeks ○ Moraxella cattarhalis *plus ≥3 of the following: □ persistent symptoms ○ recommended:
○ double worsening after episode of AVRS ○ S. aureus ○ discolored/purulent discharge □ recurrent ABRS or complications □ patients at high risk for antimicrobial resistance
□ not improved after 10 days ○ severe local pain □ when sinus surgery is contemplated □ failure of first-line treatment
□ worsen within 5-10d after initial improvement *Streptococcus is the MOST ○ temperature >38°C *worsening or no improvement after 5-7 days
COMMONLY isolated organism ○ elevation of ESR/CRP ● Sinus Radiograph ○ Amoxicillin-Clavulanate 2g BID
○ double sickening *plain films are inaccurate in a high percentage of patients ○ Doxycycline 100mg BID
□ worse again after initial recovery ○ ± partial opacification of affected sinus (mucosal swelling) ○ Levofloxacin 500mg OD
○ ± fluid level (sinus containing free pus) ○ Moxifloxacin 400mg OD
■ PSOHNS 2016 Diagnostic Criteria *Waters view: maxillary and frontal sinus
*duration: <4 weeks *Caldwell view: ethmoid air cells *Failure of second-line antibiotic treatment warrants
*acute onset of some or all of the following further work-up. This may include, but not limited to:
symptoms that fail to improve after 10 days ○ CT of the Paranasal Sinuses
○ nasal congestion ○ Sinus or meatal culture (obtained via endoscopy)
○ purulent nasal discharge ○ Immune system studies
□ anterior or posterior nasal drip
□ ± facial pain or pressure ■ Surgical Therapy
○ dental pain & ear pressure/fullness *conservative therapy should be exhausted first
○ fever, cough, fatigue *surgery: done only if no response to conservative Tx
○ hyposmia/anosmia
○ worsening within 5-10 days after initial ● Maxillary Sinus
improvement ○ maxillary sinus puncture
● Ultrasonography □ following decongestion & topical anesthesia
● Extra-Sinus Complications ○ alternative for follow-up, in children & pregnant women □ 1st approach: sharp puncture
○ periorbital edema □ less exposure to radiation *pass needle below inferior turbinate
○ displaced globe, double vision, ophthalmoplegia, ○ A-mode: acute maxillary sinusitis screening & Dx □ 2nd approach: blunt puncture
reduced Visual Acuity ○ B-mode: fluid in cavity, mucosal thickening *via natural maxillary sinus ostium
○ severe headache, frontal swelling, signs of
meningitis, neurological signs ● Computed Tomography (CT) Scan ● Frontal Sinus Empyema
*appropriate when complications of ABRS is suspected ○ beck procedure
■ Physical Examination *identify extra-sinus extension or involvement □ risk: meningoencephalitis, frontal brain abscess
*PSOHNS 2016 states that a thorough PE should ○ opacification, air-fluid level
include: ○ moderate to severe mucosal thickening
○ anterior and posterior rhinoscopy
○ inspection & palpation of frontal & maxillary sinus

LACABA 10 | P A G E
UST MEDICINE – OTORHINOLARYNGOLOGY ORAL REVALIDA ΣΒΤΦ TITANS

DISEASE & PATHOPHYSIOLOGY EPIDEMIOLOGY ETIOLOGY CLINICAL MANIFESTATIONS DIAGNOSIS TREATMENT COMPLICATIONS, PROGNOSIS
*in BLUE are recommendations of PSOHNS 2016
● Frequent Sinuses ● Chronic Rhinosinusitis ■ CRS without Nasal Polyps (PSOHNS 2016)
CHRONIC RHINOSINUSITIS *dependent sinuses *duration: ≥12 weeks ● Rhinoscopy, Endoscopy ● Medical Management
○ rhinogenic type of sinusitis lasting for ≥12 weeks ○ maxillary ○ pain (variable character) *clinical diagnosis of CRS should be supported with objective
○ CRS is divided into 2 subgroups ○ ethmoid □ feeling of pressure documentation through anterior rhinoscopy and/or nasal endoscopy (1) Intranasal Corticosteroids – primary treatment!
□ without nasal polyps (CRS w/o NP) □ persistent/recurrent headaches *Endoscopy: FIRST LINE! highly recommended! Can check the ○ Fluticasone propionate (Nasoflo) 50mcg
□ with nasal polyp (CRS w/ NP) ● Less Common Sinuses ○ postnasal drip middle turbinate! Visualize sinus drainage pathways in middle &
○ frontal ○ obstructed nasal breathing (some) superior meati (2) Nasal saline irrigation – symptomatic relief
● Nasal Polyps ○ sphenoid ○ given few minutes after intranasal corticosteroids
○ smooth, semi-translucent, pearly white to pinkish ● Definition of Chronic Rhinosinusitis ○ whitish to purulent sticky secretions ○ Flo Sinus
○ pedunculated, edematous, inflamed mucosa ○ ≥2 of the following for ≥12 weeks: ○ ± nasal polyps, lateralized middle turbinates, septal spur ○ 0.65% NaCl nasal spray, 2 sprays/nostril q6h for 1 week
○ commonly originating from ostiomeatal complex □ sudden onset nasal blockage/congestion ○ check changes of nasal septum
□ nasal discharge ○ check condition of turbinates (3) Short-term Antibiotics – for CRS in acute exacerbation
■ Common Pathogenic Mechanism □ ± facial pain or pressure □ turbinate hyperplasia ○ Amoxicillin-Clavulanate 1g BID
(1) impaired ventilation of ostiomeatal complex □ ± hyposmia or anosmia □ pneumatized middle turbinate ○ Cefuroxime 500mg BID
□ due to stenosis or obstruction by mucosal swelling □ ± endoscopic finding of: □ concha bullosa ○ Ciprofloxacin 400mg BID
□ due to intranasal anatomic changes *nasal polyps ○ check appearance of ostiomeatal unit *continue INCS
*septal deviation, septal spurs, nasal polyps *mucosal edema in middle meatus □ mucosal swelling, polyps, tumors, etc.
(2) hampered drainage of dependent sinus systems ○ other causes of impaired ventilation & drainage eg. tumors (4) Long term, Low Dose Macrolide >12 weeks
□ causes mucosal swelling that narrows passage *how to differentiate polyp from turbinate? *not advised that much in the Philippines, expensive
□ vicious cycle (narrows passage of ostiomeatal unit polyps shrink after administration of nasal ● CT Scan ○ Clarithromycin 250mg OD for 12 weeks
→ leading to recurrent bouts of inflammation corticosteroids ○ may be used to confirm diagnosis of CRS ○ Roxithromycin 150mg OD for 12 weeks
→ persistent, chronic sinusitis ○ done especially when patients with: ○ primary treatment, improve symptoms
■ PSOHNS 2016 Diagnostic Criteria □ prolonged or complicated course *continue INCS
□ failed medical management
● ≥2 of the following for ≥12 weeks: □ surgery is contemplated (preoperative planning) (5) Short Term Steroids
*should include either (a) or (b) *can be used alone or in combination with INCS or Abx
(a) nasal blockage/obstruction/congestion *benefit: very symptomatic improvement, but return after
(b) nasal discharge (ant/post nasal drip) stopping the medications
(c) facial pain/pressure ○ Prednisolone 25mg OD for 2 weeks
(d) hyposmia or anosmia ○ Methylprednisolone 0.8mkday for 1 week

● AND objective evidence of inflammatory disease ● Other Medical Therapy Options


○ mucopurulent discharge from middle meatus ○ decongestant nose drops (≤ 1 week)
○ nasal polyps ○ mucolytics: administered for supportive therapy
○ mucosal obstruction in middle meatus ○ antiallergic therapy: for allergic etiology
○ radiographic evidence showing:
□ mucosal changes within ostiomeatal complex ● Surgical Management: Endoscopic Sinus Surgery
○ within 2-3 months of INCS and no improvement
■ Other Investigations (PSOHNS 2016) ○ ESS safe for those do NOT respond to medical
SHOULD BE DISTINGUISHED FROM
management; even without polyp
Acute Exacerbation of CRS – ABRS on top of CRA ● Maxillary Aspirate C&S
○ sudden deterioration in patient’s condition, either ○ may be done in cases of acute exacerbations of CRS ■ CRS with Nasal Polyps (PSOHNS 2016)
□ worsening of baseline symptoms OR ○ establish local bacteriology & resistance in: ● Medical Management
□ development of additional symptoms □ immunocompromised 1. INCS – first line, long term treatment
Recurred ABRS □ severe infections □ Fluticasone propionate spray 200mcg BID
○ ≥4 episodes of ABRS in a year □ research purposes □ decrease polyp size, prevent postop polyp recurrence
○ WITHOUT signs & symptoms of RS between episodes 2. NSI – for symptom relied
● Investigation for Modifying Factors of CRS □ Flo Sinus
○ allergy skin testing, serum IgE – allergic rhinitis, atopy 3. short term OCS – adjunct for rapid polyp size reduction
○ aspirin sensitivity – aspirin-exacerbated respiratory diseases □ Prednisolone 25mg OD for 2 weeks
○ rhinomanometry and rhinometry – assess airflow in nasal cavity □ Methylprednisolone 16mg OD for 7 days
4. Macrolide – if poor response with INCS (grade I polyp)
□ Clarithromycin 400mg OD for 12 weeks
□ Roxithromycin 150mg OD for 8 weeks
5. Short Term Doxycycline – another option for INCS resistant
□ Doxycycline 200mg on first day, 100mg OD 20 days
6. Leukotrienes – if concomitant AR, Asthma, ASA respiratory
□ Montelukast 10mg

● Surgical Management: Endoscopic Sinus Surgery


○ if medical management fails
○ post-op INCS, NSI, topical or oral CS
□ improves polyp scire and decrease recurrence

● Indications
○ mild-moderate persistent (VAS 0-7, Grade 1-2 Polyp)
□ no improvement after 3 months medical mgt
○ severe persistent (VAS 8-10, Grade 3 Polyp)
□ no improvement after 1 month medical mgt

■ Re-Evaluation
○ Sinonasal Outcomes Test (SNOT-22) prop and postop

LACABA 11 | P A G E
UST MEDICINE – OTORHINOLARYNGOLOGY ORAL REVALIDA ΣΒΤΦ TITANS
DISEASE & PATHOPHYSIOLOGY EPIDEMIOLOGY ETIOLOGY CLINICAL MANIFESTATIONS DIAGNOSIS TREATMENT COMPLICATIONS, PROGNOSIS
*depends on extent of nasal polyposis *can be diagnosed with nasal endoscopy and CT scan
○ rarely in children ● Chronic Irritation of Mucosa *CC: progressive bilateral nasal obstruction with *gold standard of diagnosis: CT scan ■ Prognosis
NASAL POLYPOSIS ○ chronic rhinitis anosmia ○ prognosis is guarded even with modern surgical
○ edematous polypoid hyperplasia of the sinus mucosa ○ chronic sinusitis ● Rhinoscopy techniques
projecting into the nasal cavity in the form of polyps ○ nasal obstruction ○ pale-gray, semi-translucent mucosal protrusions
● Genetic ○ hyposmia/anosmia
○ headache (impaired drainage of PNS) ■ Grading of Nasal Polyps
● Other Causes ○ snoring
○ allergic rhinitis ○ rhinophonia clausa ● Grade I Polyp
○ ASA intolerance ○ frequent throat clearing (d/t postnasal drip) ○ visualize: nasal endoscopy is required
○ polyps do NOT extend beyond most anterior part of middle
*may spread to lower airways, causing turbinate
○ laryngitis with hoarseness
○ bronchitic symptoms ● Grade II Polyp
○ visualize: readily visible with nasal speculum ● Conservative Management
○ polyps extend below middle turbinate ○ intranasal corticosteroids
○ systemic steroids – may also be tried
● Grade III Polyp ○ systemic antihistamines
○ visualize: can be seen readily
○ polyps are massive, occlude entire nasal cavity ● Surgical Management
○ endoscopic sinus surgery – definitive treatment
○ intranasal polypectomy

LACABA 12 | P A G E
UST MEDICINE – OTORHINOLARYNGOLOGY ORAL REVALIDA ΣΒΤΦ TITANS
DISEASE & PATHOPHYSIOLOGY EPIDEMIOLOGY ETIOLOGY CLINICAL MANIFESTATIONS DIAGNOSIS TREATMENT COMPLICATIONS, PROGNOSIS

NASAL BONE FRACTURES ○ most frequently fractured ■ History *PE is the primary diagnosis of nasal bone fracture!!! ■ Acute Intervention ● Complications
*Source: Cummings Otolaryngology Chapter 33 nasal bone ○ Evaluate ABCDs ○ septal hematoma – clamping of nasal septum
○ fracture of the nasal bone, usually below intercanthal line ○ male > female ● Determine Cause ○ Epistaxis from sphenopalatine/ethmoid vessels ○ CSF rhinorrhea – skull base Fx
○ peak 2nd-3rd decade ○ MVA: speed, direction of impact, presence of ● Plain Radiograph □ anteroposterior packing ○ ophthalmologic complications
■ Pathophysiology restraining devices ○ 66% false positive! *ff several days of close respiratory monitoring □ destruction of lamina papyracea
○ thinner nasal bone below intercanthal line is fractured ○ assault: striking object, direction of blows ○ nasal and waters view for nasal bone Fx □ direct endoscopic cauterization or vessel ligation ○ worsening septal deviation, internasal valve collapse
ASSOCIATED INJURIES IN NASAL BONE Fx
much more easily ○ saddle nose deformity
○ hard palate instability
□ than thicker bone of nasal root above the line Le Fort Fx ● Symptoms ■ Reduction
○ open bite deformity
○ lateral forces may produce: ○ functional changes in breathing, olfaction
○ unilateral malar deformity ○ rhinorrhea ● Timing of Reduction
□ ipsilateral depressed fracture ZMC Fx
○ facial asymmetry □ bloody (epistaxis) *fibrous CT starts at 10 days to 2 weeks after injury!
□ OR outfracture the opposite nasal bone Trauma to upper portions of facial skeleton:
○ resulting in significant deviation of the entire nose □ watery, sweet/salty taste (CSF leak) *hence, advice reduction BEFORE 10 days (eg ~1 week)
○ frontal sinus fracture ○ persistent anosmia/hyposmia ○ golden period: 1hr after injury, within 24hr of injury
○ cribriform plate fracture ○ may be done after 2-3 days to allow diminution of swelling
■ Classification ○ dural tears leading to pneumocephalus
○ CSF rhinorrhea Rhinorrhea ● Septoplasty + Closed Nasal Reduction
- CSF leak: teapot sign (forward leaning/bow, then ● High Resolution UTZ ○ TREATMENT OF CHOICE for nasal fracture with septal
clear rhinorrhea), then halo sign (red center of blood, *can be used but NOT usually used! deviation of at least half the bridge, but with less severe
with clear halo on tissue paper ○ alternative to conventional radiography injuries
○ excellent for lateral nasal wall fracture
■ Physical Examination ○ disadvantage: limited scope, may miss Dx outside area of ■ Closed Reduction
*do primary survey, resuscitate interest ○ best for simple injuries, isolated, unilateral nasal bone Fx
*the proceed with secondary survey! ○ adequate for most nasal fractures

● VS – important, since trauma patient ● Finger Manipulation


○ for mild nasal bone deviation
● Nasal Examination ○ nasal bones grasped between 2 fingers
○ Inspection ○ gentle push applied laterally in opposite side of deviation
□ deformity, swelling
□ unilateral/bilateral epistaxis ● Blunt Elevation
□ periorbital ecchymosis ● CT Scan ○ Walsham Forceps
□ signs of CSF rhinorrhea ○ used to assess extent of bony injury □ locks nasal septum in place before reducing it
*teapot sign – bowing → clear rhinorrhea ○ most appropriate for evaluation of: □ good for septoplasty
*halo sign – red center, clear halo on tissue □ midfacial, orbital & frontal sinus fractures ○ Asch Forceps
○ Palpation □ each arm inserted on either side of septum
□ Brown-Gruss Provocation *positioned superiorly, underneath the septum
*grab bone below glabella, rock sideways □ lift septum out of overlap (not locked, prevent necrosis)
→ movement □ then use arms to push ends back into realignment

*REEXAMINATION 2-3 days later after swelling has *Care After Reduction
decreased, allow better appreciation of deformities ○ Anterior nasal packing – provide internal support
□ coated with antibiotic ointment
□ place superiorly within nasal cavity
□ remove 3-5 days after
□ can be left up to 1 week if (+) severe instability
○ Netcell Nasal Packing is an alternative
○ Internal Nasal Splinting
○ Avoid physical activities

■ Open Reduction
○ for more severe trauma
○ bilateral depressed fractures, with septal & cartilaginous
involvement

● Open Reduction with Fixation ± Grafting

■ Pediatric Nasal Bone Fracture

● Conservative Management
*normal nasal growth is dependent on septal cartilage
○ Intranasal Silicone Rubber Tube
□ stent nasal passage 4-5 days after manipulation
□ until edema subsides

LACABA 13 | P A G E
UST MEDICINE – OTORHINOLARYNGOLOGY ORAL REVALIDA ΣΒΤΦ TITANS

ORAL CAVITY, PHARYNX, AND LARYNGOLOGY


Approach to MOUTH, THROAT, & LARYNGEAL CHIEF COMPLAINTS
CC: SUPERFICIAL ORAL MUCOSAL LESION CC: SORE THROAT CC: HOARSENESS ■ History
*in RED are alarm symptoms that should warrant laryngoscopy after the history & physical exam!
■ Common Differential Diagnosis for REVALIDA (for exhaustive differentials, refer to AAFP 2007) ■ Common Differential Diagnosis for REVALIDA *Hoarseness: symptom of altered voice quality; reported by patient
○ Aphthous stomatitis ○ Acute Tonsillopharyngitis *Dysphonia: altered voice quality, pitch, loudness, vocal effort; assessed by clinician ● General Data
○ Herpetic gingivostomatitis (Herpes Labialis) □ Kawasaki Disease ○ occupation – professional voice user? → risk for vocal fold nodule, polyp, cyst
□ occupational (singing, public speaking, telemarketing, teaching)
□ Scarlet Fever ■ Common Differential Diagnosis for REVALIDA □ recreational (sports, umpiring, coaching)
SUPERFICIAL ORAL LESIONS ○ Infectious Mononucleosis ○ Vocal Fold Lesions
Lesion Clinical Presentation ○ Acute Epiglottitis □ Benign: Polyp, Nodule, Cyst, Varices, Granuloma, Reinke’s Edema, Tuberculosis, Papilloma ● CC, HPI: Hoarseness
○ Pseudomembranous: adherent white plaques that may be wiped off ○ Diphtheria □ Malignant: Laryngeal Squamous Carcinoma ○ onset, duration
Candidiasis ○ Erythematous: red macular lesions, often with burning sensation □ acute: infection, inflammation, injury, vocal abuse
○ Perleche: erythematous, scaling fissures at the corners of the mouth ○ Retropharyngeal Abscess ○ Laryngitis
□ chronic: benign or malignant neoplastic lesion
Recurrent Aphthous Ulcer ○ ulcer surfaced by yellowish-white pseudomembrane, erythematous halo ○ Peritonsillar Abscess □ Acute Infectious Laryngitis: Viral, Bacterial, Fungal ○ Intermittent? Continuous?
Herpetic Gingivostomatitis ○ □ Chronic Non-infectious Laryngitis: Laryngopharyngeal Reflux Disease ○ timing
Recurrent Herpes Labialis
○ prodrome (12-36 hours): itching, burning, tingling ■ History ○ Vocal Fold Paralysis □ worse early in the day → GERD, LPR
○ followed by: eruption of clustered vesicles along vermillion border → rupture, ulcer, crust □ worse later in the day → myasthenia gravis, vocal abuse
Erythema Migrans ○ migrating lesions: central erythema, surrounded by white-to-yellow elevated borders ○ identification of abnormal voice
● CC, HPI, ROS □ altered voice quality (see below)
○ reticular: white, lacy striae
Lichen Planus
○ erosive: erythema, ulcers with peripheral radiating striae; erythematous, ulcerated gingiva ○ OLDCARTS of odynophagia □ pitch – pitch breaks, vocal range
○ fever, malaise □ loudness – vocal fatigue (loudness decrease throughout the day)
□ voice effort – reduced vocal endurance, sensation of increased effort
ORAL LESIONS APPEARING AS MASS ○ extreme fatigue, anorexia → Infectious mononucleosis
Lesion Clinical Presentation ○ dysphagia → peritonsillar abscess, acute epiglottitis ● Associated & Aggravating Factors
Palatal & Mandibular Tori ○ bon protuberance of palate or lingual aspect of mandible ○ hoarseness ○ Hx misuse or abuse of voice → vocal fold nodule, polyp, cyst
Pyogenic granuloma ○ rapidly growing, red, lobulated mass ○ symptoms of URTI (cough, colds), recent ENT infections → abscesses ○ Hx URTI → acute laryngitis
Mucocele ○ bluish, fluctuant mucosal swelling, often with history of rupture ○ difficulty of breathing, stridor → acute epiglottitis ● Associated Symptoms & ROS
Fibroma ○ firm, pink, smooth-surfaced nodule ○ painful cervical lymph nodes → Acute tonsillopharyngitis, Kawasaki, Scarlet, PT & RP Abscesses ○ weight loss → TB, SCCA
Leukoplakia ○ white patch that DOES NOT wipe off (unlike Candidiasis)
○ previous episodes of sore throat, recurrent? ○ heartburn → LPR
Erythroplakia ○ red patch without obvious cause
SCCA ○ non-healing ulcer or mass
□ for paradise criteria ○ difficulty of breathing (dyspnea, stridor) → subglottic extension of SCCA, large papilloma or lesions
○ sore throat, odynophagia, → laryngitis
□ risk factor for peritonsillar abscess, retropharyngeal abscess ○ dysphagia → large lesions involving supreglottis
○ drooling, change in voice (muffling) → peritonsillar abscess, retropharyngeal abscess ● Vocal Fold Lesions DDx ○ non-productive cough → if chronic, think tuberculosis
○ RUQ or mid-epigastric pain → infectious mononucleosis
Lesion Etiology Clinical Manifestation Laryngoscopy ● PMHx
● bilateral ○ acromegaly, hypothyroidism
○ Hx of ingestion of foreign body (eg fish bones) → check if odynophagia is d/t foreign body Nodules
vocal abuse ● hoarseness
○ fibrovascular thickening at midline (junction bet ant
female, 18-39 years old ○ ○ tuberculosis – ask for close contacts
○ Hx hyperacidity, GERD, PUD → assess for LPR ½ and post 2/3)
○ Hx of hyperacidity/GERD/PUD → LPR
vocal abuse, trauma, &
○ Hx snoring, sleep apnea → check if tonsillar condition affects OSA Cyst anticoagulant
● hoarseness

● unilateral
○ encapsulated subepithelial mass
○ previous neurosurgery, neck, thyroid, thoracic surgeries → vocal fold paralysis
○ Hx of exposure to close contact with sore throat (hemorrhagic cyst)
allergy, tobacco & other irritants, ● hoarseness ● unilateral, can be bilateral
○ previous intubation → trauma, may lead to vocal fold granuloma
Polyp vocal abuse ○ ○ well-defined sessile or pedunculated midpoint mass
● Medication History – may be exacerbating factors!!!
■ Physical Examination Reinke’s
GERD, LPR, tobacco & other
irritants, vocal abuse ● hoarseness
● bilateral (commonly) ○ ACE inhibitors – cough
○ proliferation of superficial mucosa over entire length
Edema female, middle-aged (40-50) or ○ frequent throat clearing
of 1 or both vocal folds ○ Antihistamines, diuretics, anticholinergics – drying effect on mucosa
older age (>60yo)
● Vital Signs ○ may be asymptomatic
○ antipsychotics – laryngeal dystonia
● unilateral ○ bisphosphonates – chemical laryngitis
○ vital signs (RR, temperature) Granuloma
traumatic mucosal disruption,
endotracheal intubation, LPR
● hoarseness
○ discomfort at level of throat
○ hypertrophic inflammation, exophytic with narrow
○ danazol, testosterone – sex hormone alteration
○ signs of respiratory distress → acute epiglottitis ○ dyspnea (if bulky)
base, usually in arytenoid region
○ inhaled corticosteroids – mucosal irritation; candida or fungal laryngitis
● hoarseness
○ airway obstruction ● asymmetric
○ warfarin, thrombolytics – vocal fold hematoma
Papilloma HPV 6, 11; sexual transmission
● Skin Examination ○ stridor ○ benign cauliflower-like lesions
○ weak cry (in babies) ● Current Health & Risk Factors
○ exanthems? ● hoarseness ○ alcohol intake, smoking → risk factor for SCCA
● asymmetric
○ scarlet fever? check for goosebump sandpaper-like rashes, pastia’s lines Tuberculosis MTB, close contacts
○ low grade fever
○ weight loss
○ ulcerofungative, polypoid ○ eating habits → refluxogens for LPR such as spicy foods, fatty foods etc.
○ Kawasaki disease? check for polymorphous rashes, erythema of palms and soles ○ night sweating
○ Infectious mononucleosis? Check for maculopapular rashes Laryngeal smoking, excessive alcohol
● hoarseness – m/c (glottic)
○ dysphagia (supreglottic)
● asymmetric ■ Physical Examination
FHx of malignancy ○ friable, whitish plaque, any portion, can extend
SCCA HPV, LPR
○ dyspnea (subglottic)
contralaterally if at anterior commissure
○ (+) neck node ● Vital Signs
● Eye Exam ○ RR – evaluate respiration with phonation & respiratory pattern
○ check for conjunctival injection → Kawasaki? ○ temperature – fever in infectious causes such as laryngitis, TB

● Mouth & Throat Exam, Laryngoscopy ● Complete Head & Neck Examination
○ Inspection: motion & symmetry of face, oral cavity, head & neck
○ trismus → peritonsillar absess
○ Neck: check for neck mass, cervical lymphadenopathies → laryngitis, metastasis from SCCA
○ strawberry tongue → Kawasaki, scarlet fever Nodules Cyst Polyp Reinke’s Edema
○ palate ● Dysphonia Evaluation
○ tonsils: exudates, size
○ GRBAS Scale
□ Grade 0 – in the tonsillar fossa □ each parameter quantified on 4-point scale
□ Grade 1 - ≤25% □ 0 = normal, 1 = mild, 2 = moderate, 3 = severe
□ Grade 2 - ≤50%
□ Grade 4 - ≤75% Granuloma Papilloma Tuberculosis Laryngeal SCCA Parameter Description
□ Grade 5 - >75% Grade overall degree of voice abnormality
○ posterior pharyngeal wall – erythema? Assess if there is pharyngitis ■ Approach to Dysphonia (AAO-HNS 2018) Roughness perceived irregularity in voicing source
Breathiness audible air escape in voice
○ check for pseudomembrane → diphtheria
Asthenia voice weakness
○ peritonsillar swelling, deviated uvula → peritonsillar abscess Strain perception of excessive vocal effort
○ bulging posterior pharyngeal wall → retropharyngeal abscess
○ Altered Voice Quality
● Neck Examinations □ frequency or pitch – should be appropriate for age, gender, body habitus
○ bull neck appearance → diphtheria □ volume or vocal intensity – should be appropriate for conversational speech
□ timing of voice onset – speech breaks, overall rhythm
○ cervical lymphadenopathies → Acute TN, Scarlet, Kawasaki □ voice effort
○ neck stiffness → retropharyngeal abscess
Vocal Quality Suggested Diagnoses
● Abdominal Examination Inflammatory arthritis, spasmodic or functional dysphonia
Breathy
vocal fold mass, vocal fold paralysis
○ check for hepatosplenomegaly → infectious mononucleosis
Halting, Strangled spasmodic dysphonia
Hparse, Husky, Muffled, Nasal-
Parkinson Disease
Sounding
GERD, hypothyroidism, LPR, leukoplakia, muscle tension dysphonia,
Low Pitched
Reinke edema, vocal fold edema, age-related vocal atrophy in women
Scanning
multiple sclerosis
speech & dysarthria
Soft (Loss of Volume) vocal fold paralysis, Parkinson disease, age-related vocal atrophy
Spoken voice lost, but whispered
conversion aphona
voice maintained
Strained GERD, LPR, muscle tension dysphonia, spasmodic dysphonia
Strained, effortful phonation muscle tension dysphonia
Thick, deep voice and slowed
acromegaly
speech
muscle tension dysphonia, myasthenia gravis, Parkinson disease
Vocal fatigue
vocal abuse, age-related vocal atrophy

LACABA 14 | P A G E
UST MEDICINE – OTORHINOLARYNGOLOGY ORAL REVALIDA ΣΒΤΦ TITANS
SUPERFICIAL ORAL MUCOSAL LESION

DISEASE & PATHOPHYSIOLOGY EPIDEMIOLOGY ETIOLOGY CLINICAL MANIFESTATIONS DIAGNOSIS TREATMENT COMPLICATIONS, PROGNOSIS
*systemic disorders should be suspected in presence of
○ 5-21% prevalence ● Possible Causes ■ Physical Examination features that suggest systemic background ■ Idiopathic Aphthous Ulcer
RECURRENT APHTHOUS STOMATITIS ○ Hematinic deficiency ■ Oral Ulcer
○ aka “Canker Sores” □ iron, folate, vitamin B12 ○ recurrent, painful, solitary or multiple ulcers ■ Diagnosis of RAS ● Topical corticosteroids – given after meals
○ precise pathogenesis unknown ○ GI Causes ○ covered by white-to-yellow pseudomembrane ○ based only on history and clinical features ○ Fluocinonide (0.05% gel) 2-4x per day
□ Celiac Disease ○ surrounded by erythematous halo ○ no specific tests are available ○ Triamcinolone acetonide (0.1% paste) 2-3x per day
□ IBD, Crohn Disease
○ Behcet’s Syndrome *Predilection of Oral Ulcer: Non-keratinizing ■ Tests to Exclude Systemic Disorders ● Chlorhexidine Mouthwash
○ HIV Infection ○ labial mucosa ○ Chlorhexidine gluconate 0.2% mouthwash BID
○ PFAPA Syndrome ○ buccal mucosa ● Hematinic Deficiency □ rinse mouth BID with 10mL for 1 minute
○ ventral tongue ○ CBC □ decrease severity but does not reduce incidence
*smoking is associated with lower ○ Red Cell Indices
prevalence ○ Iron Studies ● Amlexanox 5% paste (Aphthasol)
○ RBC Folate Assay ○ 4x per day following oral hygiene after meals & bedtime
■ History ○ Serum Vitamin B12 ○ may promote healing and lessen pain
*confirm recurrence
*rule out associated conditions, recurring ● HIV *Thalidomide may be used is off-label and is reserved for
intraoral herpes infections ○ WBC with Differential, CD4 count severe, constantly recurring RAS such as those associated
○ screen using: EIA with HIV infection
● HPI
○ confirm with: HIV Western Blot
○ painful oral ulcer
○ localize – usually anterior 1/3, involving only
■ Management of Underlying Causes
non-keratinizing mucosa ● Celiac Disease
○ check for recurrence ○ Serum antoendomysium antibody ● Hematinic Deficiency – provide supplementation
○ Transglutaminase assay
● ROS ■ Clinical Forms ● Behcet’s Disease
*thorough history is essential; ROS can assist in *three: minor, major, herpetiform ● IBD ○ Azathioprine - for eye disease
determining whether ulcer is idiopathic or ○ colonoscopy with biopsy & histopathology ○ Steroids, Sucralfate - for oral & genital ulcers
related to a systemic inflammatory process ● Minor Aphthae – most common ○ pANCA (UC), ASCA (CD) ○ Colchicine – erythema nodosum/arthritis (male), genital
○ Behcet’s: red eye, skin & genital ulcers
○ rounded, well-demarcated, superficial ulcers (female)
○ Celiac Disease: diarrhea with gluten ingestion
○ IBD: crampy abdominal pain, chronic
○ single or multiple ● Behcet’s Disease
diarrhea, hematochezia, tenesmus, ○ diameter: <1cm ○ Diagnostic Criteria: recurrent oral ulcer (>3/yr) + 2 of the ● HIV
○ HIV: recurrent previous infections, severe ○ heals by 10-14 days without scarring following: ○ refer to ID for ART
infections *located in anterior 1/3 of oral cavity □ recurrent genital ulcer >3/yr
○ PFAPA: history of periodic fever, aphthous □ eye: red eye, uveitis ● IBD
stomatitis, pharyngitis, and adenitis ● Major Aphthae □ skin: erythema nodosum, ulcer, purpura, folliculitis ○ refer to Gastroenterology, 5-ASA
○ deeper ulcerations □ pathergy (papules on NSS infusion)
● Tertiary History ○ diameter: >1cm (large) ○ Pathergy Test
○ food Hx: check for hematinic deficiencies
○ heals by 2-4 weeks with scarring □ small red bump where NSS was infused 1-2d before

*Source: Probst *Source: Probst, MIMS Philippines


■ Etiologic Agent ● Clinical Diagnosis ■ Complications (Probst)
ORAL HSV INFECTIONS ○ HSV1 – m/c oral mucosa ■ Herpetic Gingivostomatitis ○ generally based on history and clinical examination ■ Primary Herpes Simplex Infection
○ HSV infection that predominantly affect oral mucosa ○ HSV2 – m/c genital region ○ initially: flu-like symptoms ● Herpes Impetiginatus – feared complication!
□ fever, lethargy, with regional lymphadenitis ● Viral Identification ● Systemic Acyclovir ○ secondary Bacterial Superinfection
■ Herpetic Gingivostomatitis ● MOT ○ followed by bullae on oral mucosa *ordinarily, there is no need for costly methods of viral ○ Acyclovir 200mg 5x/day q4hrs (omit nighttime dose) for 5-10 ○ S. aureus, Streptococcal infections
○ primary HSV infection usually in early childhood ○ contact or droplet identification such as: EM, IF microscopy, PCR days ○ heals by scarring
○ appears as aphthous stomatitis! ○ superficial skin injuries ○ Tzanck Smear – cytologic exam, classic giant cells *child <2yo half the adult dose
(occasional) *child ≥2yo same as adult dose ● Postherpetic Exudative Erythema Multiforme
■ Herpes Labialis ○ skin lesions + ulcerative lesions on mucous membranes
○ due to reactivation of previous HIV infection ■ Triggers for Reactivation ■ Recurrent Herpes Labialis of mouth, lips, and genitals
○ reactivation occurs in response to (see etiology) *particularly in herpes labialis
○ physical exertion ● Topical Antiseptics
○ UV radiation ○ prevent superinfection
○ febrile infection
○ emotional stress ● Topical Antiviral – for severe forms
○ pregnancy ■ Recurrent Herpes Labialis ○ Acyclovir 5% cream 5x/day q4 hours for 5-7 days
○ prodrome: 12-36 hours □ child: same as adult dose
□ itching, burning, tingling ○ Acyclovir mucoadhesive buccal tab 1 tab (50mg)
○ followed by eruption of clustered vesicles □ single dose to upper gum 1hr after onset if prodrome
□ along vermillion border □ give before appearance of any herpes labialis
□ rupture, ulcerate, and crust
■ Suppression of Recurrent Herpes Simplex

● Systemic Acyclovir
○ Acyclovir 400mg BID q12 hours
□ child 12-17yo same as adult dose

LACABA 15 | P A G E
UST MEDICINE – OTORHINOLARYNGOLOGY ORAL REVALIDA ΣΒΤΦ TITANS
SORE THROAT

DISEASE & PATHOPHYSIOLOGY EPIDEMIOLOGY ETIOLOGY CLINICAL MANIFESTATIONS DIAGNOSIS TREATMENT COMPLICATIONS, PROGNOSIS
○ children ● Bacterial ● Bacterial Tonsillopharyngitis *request for a CBC to check for leukocytosis & neutrophilic predominance ● Conservative Therapy ● Complications of Acute Tonsillopharyngitis
ACUTE TONSILLOPHARYNGITIS ○ adolescents ○ Streptococcus pyogenes ○ sudden onset sore throat
*ESR and CRP can also be requested although not absolutely necessary
*based on my CENTER Score of 1-3, I’d like to request for a RADT ○ adequate fluid intake ○ rheumatic fever
○ infection of the pharynx, palatine tonsils, or both ○ Staphylococcus aureus ○ headache, nausea, vomiting, abdominal pain ○ warm water gargle ○ rheumatic heart disease
○ Haemophilus influenzae ○ severe pain on swallowing, radiating to ear ■ CENTOR Score ○ antibiotics for streptococcal infection ○ PSGN
○ Bacteroides fragilis ○ muffling of speech d/t oropharyngeal swelling ○ for diagnosis of GABHS Pharyngitis □ Amoxicillin 40-50 mkday in 3 divided doses x10 days ○ otitis media
○ NO cough □ Amoxicillin 500mg BID ○ meningitis
● Viral ○ ± scarlantiform rash Criteria Points ○ antipyretics for fever: Paracetamol 10-15 mkdose q4h ○ pneumonia
○ Rhinoviurs ○ ± high fever >38°C in the past 24 hours Clinical Findings ○ agents for sore throat relief ○ abscess formation
○ Coronavirus ○ tender cervical lymph nodes absence of cough +1 □ Kamillosan spray 2 sprays TID or □ retropharyngeal abscess
○ Adenovirus ○ enlarged red tonsils ± tonsillar swelling/exudates swollen & tender anterior cervical nodes +1 □ Benzydamine HCl (Difflam) 3mg/lozenge TID □ peritonsillar abscess
temperature >38°C +1
○ Herpes virus
tonsillar exudates or swelling +1
● Viral Tonsillopharyngitis Patient’s Age ANTIBIOTIC THERAPY ● Tonsilloliths
○ cough & coryza 3 to 14 years +1
○ hoarseness 15 to 44 years 0 ■ Management Based on CENTOR Scoring
○ diarrhea 45 years and older -1
○ scleral conjunctival infection Score Risk Probability Action
-1 to 0 Low 1% no further testing or treatment
○ pharyngeal ulcerations ● Diagnostic Tests throat culture ± RADT
1 to 3 Moderate 10-35%
○ viral exanthems Tx based on culture result
○ GOLD STANDARD but takes a long time ≥4 High 51% consider empiric treatment
Throat Culture ○ diagnostic standard (sensitivity 90-95%)
○ results after 5 days
Rapid Antigen ○ earlier treatment, symptom improvement
■ Antibiotic Options for GABHS Pharyngitis
Detection Test (RADT) ○ results within 5-10 minutes
○ confirm previous infection
Streptococcal
○ in suspected acute PSGN or Rheumatic Fever
● Indications
Antibody Titer ○ acute symptom relief
○ ↑1-2 weeks then peak 3-5 weeks after infection
○ prevention of complications
■ Brodsky Tonsil Grading ○ reduced communicability ● Peritonsillar Abscess
○ reduce incidence of acute rheumatic fever ○ ipsilateral palatal edema
○ contralateral uvular deviation
Drug Dosage for Children Duration
12.25-22.5 mg/kg 2x/day
Amoxicillin 10 days
10-13.3 mg/kg 3x/day
Erythromycin 30-50 mg/kg/day in 2-4 divided doses 10 days
Cephalexin 25-50 mg/kg/day in 2-4 divided doses 10 days

Drug Dosage for Adults Duration


Amoxicillin 250mg TID or 500mg BID 10 days
Grade 0 tonsil within tonsillar fossa Erythromycin 400mg QID or 800mg BID 10 days
Grade 1 just outside tonsillar fossa; occupy ≤25% oropharyngeal width Cephalexin 500mg BID 10 days
Grade 2 occupy 26 to 50% (≤50%) of oropharyngeal width
Grade 3 occupy 51 to 75% (≤75%) of oropharyngeal width
Grade 4 occupy >75% oropharyngeal width TONSILLECTOMY

● Absolute Indication
DIFFERENTIAL DIAGNOSIS ○ suspected malignancy (tonsil ulceration, unilateral swelling)
○ tonsillitis, fatigue, anorexia ○ obstructive sleep disordered breathing
○ moderate fever (38-39°C)
Infectious
○ swollen tonsillar, nuchal, axillary, inguinal LN ■ Paradise Criteria
Mononucleosis
○ hepatosplenomegaly, RUQ/mid-epigastric pain
○ Dx: monospot test, liver enzymes, CMV Criterion Definition
Minimal Frequency 7 or more episodes in the preceding year OR
○ high grade fever, swallowing difficulty of Sore Throat 5 or more episodes in each preceding 2 years OR
Diphtheria ○ bull neck (thickened neck 2° lymphadenopathy) Episodes 3 or more episodes in each preceding 3 years
○ pseudomembrane formation (airway obstruction) PLUS
temperature >38.3°C OR
○ inspiratory stridor Clinical Features
cervical lymphadenopathy (tender or >2cm) OR
Epiglottitis ○ severe respiratory distress tonsillar exudates OR
positive culture for GABHS
○ cherry red epiglottis antibiotics had been administered in conventional
Treatment
Retropharyngeal ○ complication of bacterial pharyngitis dosage for proved or suspected streptococcal episodes
each episode has been substantiated in clinical record
Abscess ○ bulging in posterior pharyngeal wall OR
Peritonsillar ○ (+) unilateral bulging above & lateral to tonsil Documentation if not fully documented, subsequent observance by
clinician of 2 episodes of throat infection with patterns of
Abscess ○ (+) contralaterally deviated uvula frequency & clinical features consistent with initial history

*provide option for tonsillectomy if:


□ recurrent sore throat, previous episodes documented
□ recurrent sore throat, 2 subsequent episodes documented
□ recurrent sore throat, PFAFA, peritonsillar abscess, previous ABx

LACABA 16 | P A G E
UST MEDICINE – OTORHINOLARYNGOLOGY ORAL REVALIDA ΣΒΤΦ TITANS
DISEASE & PATHOPHYSIOLOGY EPIDEMIOLOGY ETIOLOGY CLINICAL MANIFESTATIONS DIAGNOSIS TREATMENT COMPLICATIONS, PROGNOSIS

INFECTIOUS MONONUCLEOSIS adolescents & young adults ● Epstein-Barr Virus (EBV) ● History ○ monospot test ○ rest
○ aka Pfeiffer’s glandular fever, Kissing disease (15-24 yo) ○ MOT: saliva, mucus, and tears ○ tonsillitis – common initial or cardinal symptom ○ liver enzyme ○ throat soothing measures
○ systemic: fatigue, anorexia ○ CMV ○ acetaminophen/ibuprofen
○ moderate temperature elevation (38-39°C) ○ low energy/impact activity (prevent splenic rupture)
○ severe pain on swallowing, headache, limb pains ○ gradual ↑ activity
○ pain in RUQ and mid-epigastric and/or LUQ

● Physical Examination
○ swollen tonsillar, nuchal, axillary & inguinal lymph
nodes
○ hepatosplenomegaly

DIPHTHERIA controlled due to active ● Corynebacterium diphtheriae ● History ○ (+) grayish-yellow pseudomembranes ○ diphtheria antitoxin (200-1000IU/kg body weight) IV or IM
○ C. diphtheriae produces special endotoxin that causes immunization ○ MOT: droplet inhalation, skin-to- ○ begins with fever & mild swallowing difficulties → firmly adherent to the tonsils may spread to the palate ○ Penicillin G
epithelial cell necrosis and ulcerations skin contact ○ malaise, headache, nausea and pharynx
→ when removed, underlying tissue bleeds
● Physical Examination ○ smear findings
○ bull neck (due to severe lymphadenopathy;
causing thickening of neck)
○ pseudomembrane formation (causing airway
obstruction & dyspnea)

LACABA 17 | P A G E
UST MEDICINE – OTORHINOLARYNGOLOGY ORAL REVALIDA ΣΒΤΦ TITANS
TONSILLAR HYPERTROPHY & OBSTRUCTIVE SLEEP APNEA SYNDROME

DISEASE & PATHOPHYSIOLOGY EPIDEMIOLOGY ETIOLOGY CLINICAL MANIFESTATIONS DIAGNOSIS TREATMENT COMPLICATIONS, PROGNOSIS
■ Recurrent Tonsillitis ■ Friedman Tonsil Grading ■ Tonsillectomy ● Complications
CHRONIC HYPERTROPHIC TONSILLITIS ○ sudden onset sore throat *indications are based in AAO-HNS recommendations ○ recurrent tonsillitis
○ chronic enlargement of the tonsil uncomplicated by ○ headache, nausea, vomiting, abdominal pain ○ obstructive sleep apnea
inflammation ○ severe pain on swallowing, radiating to ear ● Absolute Indications
○ muffling of speech d/t oropharyngeal swelling ○ enlarged tonsils that cause:
● Tonsils ○ NO cough □ upper airway obstruction
○ independent organs constructed by MALT ○ ± scarlantiform rash □ severe dysphagia
○ regulate immune response in internal environment ○ ± high fever >38°C in the past 24 hours □ sleep disorders
○ tonsil involution usually occurs 8-10yo ○ tender cervical lymph nodes □ or cardiopulmonary complications
○ enlarged red tonsils ± tonsillar swelling/exudates ○ peritonsillar abscess that is:
● Blood Supply of the Palatine Tonsils □ unresponsive to medical management & drainage
○ ECA ■ Obstructive Sleep Apnea *unless if surgery is performed during acute stage
→ Facial a → Tonsillar, Ascending Palatine ○ tonsillitis resulting in febrile convulsions
→ Lingual ● Associated Symptoms ○ tonsils requiring biopsy to determine tissue pathology
→ Ascending Pharyngeal → Descending Palatine ○ excessive daytime sleepiness □ suspected malignancy if:
○ impaired cognition *tonsil ulceration, unilateral swelling
○ mood disorders
○ insomnia ● Relative Indications
○ hypertension ○ ≥3 tonsil infections/year despite adequate medical therapy
○ ischemic heart disease or history of stroke ○ persistent foul taste or breath
■ Brodsky Tonsil Grading □ d/t chronic tonsillitis NOT responsive to medical therapy
● Physical Findings Suggestive of SDB ○ chronic or recurrent tonsillitis
○ obese □ in a chronic carrier who is
○ large neck □ NOT responding to β-lactamase resistant antibiotics
□ men: >42cm (17”) ○unilateral tonsil hypertrophy that is presumed neoplastic
□ women: >36cm (16”)
○ retrognathia or micrognathia ■ Paradise Criteria
□ cannot fit 3 fingers under the lip
○ crowded airway (check by Mallampati Score) Criterion Definition
Minimal Frequency 7 or more episodes in the preceding year OR
○ enlarged tonsils (in children, Brodsky Grading) Grade 0 tonsil within tonsillar fossa
of Sore Throat 5 or more episodes in each preceding 2 years OR
Grade 1 just outside tonsillar fossa; occupy ≤25% oropharyngeal width
○ high arched palate Grade 2 occupy 26 to 50% (≤50%) of oropharyngeal width
Episodes 3 or more episodes in each preceding 3 years
PLUS
□ Edward’s syndrome Grade 3 occupy 51 to 75% (≤75%) of oropharyngeal width
temperature >38.3°C OR
□ Treacher Collins syndrome Grade 4 occupy >75% oropharyngeal width
Clinical Features
cervical lymphadenopathy (tender or >2cm) OR
tonsillar exudates OR
○ nasal deformities, septal deviation positive culture for GABHS
antibiotics had been administered in conventional
Treatment
dosage for proved or suspected streptococcal episodes
each episode has been substantiated in clinical record
OR
Documentation if not fully documented, subsequent observance by
clinician of 2 episodes of throat infection with patterns of
frequency & clinical features consistent with initial history

*provide option for tonsillectomy if:


□ recurrent sore throat, previous episodes documented
□ recurrent sore throat, 2 subsequent episodes documented
□ recurrent sore throat, PFAFA, peritonsillar abscess, previous ABx

LACABA 18 | P A G E
UST MEDICINE – OTORHINOLARYNGOLOGY ORAL REVALIDA ΣΒΤΦ TITANS
DISEASE & PATHOPHYSIOLOGY EPIDEMIOLOGY ETIOLOGY CLINICAL MANIFESTATIONS DIAGNOSIS TREATMENT COMPLICATIONS, PROGNOSIS
*see STOP-Bang ■ Predisposing Risk Factors ● Associated Symptoms in OSA MANAGEMENT OF OSAS IN ADULTS MANAGEMENT OF OSAS IN ADULTS ● Complications
OBSTRUCTIVE SLEEP APNEA ○ male, obese patients, >50yo ○ Age (40 to 70 years) ○ Cognitive impairment ■ Severity & Assessment ○ most serious complication: SLEEPINESS (risk for self & others)
○ Commercial motor vehicle driver ○ higher severity OSA = higher mortality risk
SYNDROME ○ Family history of OSA ○ Daytime sleepiness ○ obstructive apneas & hypopneas increase risk for stroke
○ disorder of breathing during sleep ○ Male sex ○ Fatigue ○ people with HTN are of higher risk to acquire OSA
○ characterized by prolonged partial airway obstruction ○ Obesity (BMI ≥30 kg per m2) ○ Insomnia
○ and/or complete obstruction that disrupts normal ○ Postmenopausal woman not taking ○ Loud snoring Complications Odds Ratio (95% CI)
hormone Hypertension 1.5
ventilation during sleep & normal sleep patterns ○ Mood disorders Simple Snoring 3.3
○ Preoperative for bariatric surgery
○ Retrognathia ○ Sleep episodes during wakefulness RDI 5-15 (mild) 3.3
■ Stages of Sleep ○ Waking up gasping for air, choking, or holding Mallampati Airway Classification – tongue is protruded as far as possible
RDI 15-30 (moderate) 4.8
RDI >30 (severe) 6.6
*4-6 cycles of NREM-REM per night with progressively increasing REM ■ Upper Airway Risk Factors one's breath Class I soft palate and entire uvula visible Ischemic Heart Disease 1.9
○ short N1 followed by long N2 (50% of time spent) Class II soft palate & portion of uvula visible (1) Risk Factor Assessment & Modification Myocardial Infarction 5.5
○ N3 is deepest sleep (stage where you feel rested) ○ Obesity Class III soft palate visible (may include base of uvula) Sudden Death 4.1
○ REM achieved in 60-90 mins (dreams, atonia, significant in OSA d/t ● Cardiovascular Comorbidites Class IV soft palate not visible
○ Jaw Malformation Risk Factor Action Stroke 3.1
loss of muscle tone!!!) □ Rheumatoid Arthritis ○ hypertension Non-Anatomic
□ Congenital Craciofacial Anomaly ○ ischemic heart disease Obesity
WEIGHT LOSS (decrease in weight improves AHI) ● OSA & Stroke
*10% weight loss = great improvement from sleep apnea
○ Floppy Epiglottis ○ previous stroke Alcohol avoid alcohol (alcohol relaxes upper airway)
○ med snorers are 2x to have stroke
○ Paralysed Vocal Cord Drugs stop use of muscle relaxants & tranquilizers (decrease airway) ○ heavy snorers are 4x to have stroke
○ Upper Airway Tumor ● Physical Findings Suggestive of SDB Medical treat medically (hypothyroid, acromegaly)
○ Enlarged Soft Palate Positional
□ Cleft Palate Repair
○ obese Snoring
positional training (sleep on either side, not supine)

○ Enlarged Tonsils and Adenoid ○ large neck Friedmann Staging System in OSA – tongue is in the natural (neutral) position Nasal
treat medically, try nasal dilator
Obstruction
□ Most common among kids □ men: >42cm (17”) a FTP I visualize entire UVULA and TONSILS
Anatomic
b FTP IIa visualize UVULA but tonsils only partially seen
○ Enlarged Tongue □ women: >36cm (16”) Nasal try nasal dilator
FTP IIb visualize COMPLETE SOFT PALATE down to base of
□ Down’s Syndrome ○ retrognathia or micrognathia c uvula Pharynx see below
● Sleep & Aging □ Acromegaly but uvula and tonsils not seen Retrognathia,
○ N3 (slow wave, deepest sleep) decreases as we age □ Hypothyroidism □ cannot fit 3 fingers under the lip FTP III visualization of SOME OF SOFT PALATE Micrognathia
see below
d
○ WASO (wake after sleep onset) become more frequent □ Amyloidosis ○ crowded airway (check by Mallampati Score) but distal soft palate is eclipsed
○ Nasal Obstruction ○ enlarged tonsils (in children, Brodsky Grading) e FTP IV visualization of HARD PALATE ONLY ● Obesity Treatment
■ Definition of Terms: Apnea & Hypopnea □ Deviated Septum ○ WEIGHT LOSS IS MOST EFFECTIVE & IMPORTANT!!!
○ high arched palate
□ Chronic Rhinitis ■ Screening & Identifying SDB ○ start pharmacotherapy if patient is already obese
● Obstructive vs Central Apnea □ Edward’s syndrome
□ Antrochoanal Polyp
○ obstructive: (+) brain signal (-) airflow (+) chest, abdominal movements □ Treacher Collins syndrome ● STOP-Bang Questionnaire BMI Category
Treatment
○ central: (-) brain signal (-) airflow (-) chest, abdominal movements ○ nasal deformities, septal deviation ○ validated screening tool for OSA (85% sensitivity) ≥25 ≥27 ≥30 ≥35 ≥40
Diet,
● Obstructive Hypopnea vs Obstructive Apnea Exercise, (+) (+) (+) (+) (+)
Behavior
○ hypopnea: shallow breathing, asynchronous, paradoxical breathing
Pharma comorbid (+) (+) (+)
○ apnea: disappearance of breathing ≥10 seconds Surgery comorbid (+)

■ Definitions of SDB ● Bariatric Surgery


○ invasive, done for very or morbidly obese patients
Apnea cessation of airflow for ≥10 seconds
Hypopnea reduction in airflow w/ resultant desaturation of ≥4%
○ open stomach, narrows down size of stomach to accommodate less food
reduction in airflow w/ resultant arousal ○ supplements are needed if with bypass
RERA
BUT not meeting desaturation criteria for hypopnea
AHI average frequency of apnea & hypopnea events per hr of sleep (2) Continuous Positive Airway Pressure (CPAP)
RDI
average frequency of apnea, hypopnea, & RERA per hr of sleep ○ GOLD STANDARD for treatment of OSA
obtained by polysomnography
○ mechanism: push (+) pressure resulting to increase in airway caliber
○ must be used at least 4 hours at night time
■ Patterns of Breathing during Sleep ○ titration studies: determine best pressure suited for the patient
*the greatest likelihood that you will have OSA is stage R!!! Low Risk 0-2 ○ advantage: disappearance of desaturation, improve BP
Intermediate Risk 3-4
○ decreased respiratory drive High Risk ≥5 (3) Oral Appliance
○ minimized non-chemical respiratory input ○ slowly advances the jaw forward, increasing size for airway
NREM
○ breathing regulated by metabolic respiratory ctrl ■ Diagnosis of SDB ○ only for young, ↓BMI, smaller neck circumference, ↓AHI, positional OSA
○ ↓VE + slight ↑PaCO2 *also needs complete teeth (where appliance will hold on to)
○ irregular respiratory drive ● Polysomnography
○ reduced ventilatory response to chemical/ ○ can quantify apnea-hypopnea index (4) Surgical Approach
mechanical stimuli (even if ↓PaCO2, no ↑RR) ○ type I sleep study: gold standard, done in the lab ○ tongue reduction surgery
REM
○ short periods of central apnea ○ type IV: portable and NOT ideal ○ lase midline glossectomy
○ ↓intercostal & accessory muscle activity ○ parameters of polysomnography: ○ lingualplasty
○ ↓thoraco-abdominal coupling ○ upper airway bypass
EEG α waves (N1, R), β spindles (N2), slow waves (N3) ○ tracheostomy (invasive, 1st treatment of OSA in history)
■ Pathophysiology EOG eye movement, determine REM ○ bariatric surgery, gastric bypass, jejunoileal bypass
EMG muscle tone, determine REM atonia, bruxism, grinding ○ uvulopharyngeoplasty (UPPP) – removal of uvula & pharynx by ENT,
EKG arrythmias
problem is that it eventually narrows down airway; hence, NOT standard
Microphone snoring, sleep talking
PR Aitflow shallowing/disappearance of breathing
○ radiofrequency ablation – ablate area to change size of airways
O2 Sat assess desaturation following airway obstruction
Leg restless leg syndrome, leg kicking following airway obstruction MANAGEMENT OF OSAS IN CHILDREN
○ CPAP, Weight Loss
● Apnea-Hypopnea Index (AHI) ○ Intranasal Corticosteroids (INCS)
○ classify severity of OSA syndrome □ for mild OSAS in whom adenotonsillectomy is contraindicated
○ AHI ≥ 5 + associated symptoms = OSA □ also for mild postoperative OSAS

Severity AHI ● Adenotonsillectomy


Mild 5-14
Moderate 15-29
Severe ≥30

LACABA 19 | P A G E
UST MEDICINE – OTORHINOLARYNGOLOGY ORAL REVALIDA ΣΒΤΦ TITANS
HOARSENESS

DISEASE & PATHOPHYSIOLOGY EPIDEMIOLOGY ETIOLOGY CLINICAL MANIFESTATIONS DIAGNOSIS TREATMENT COMPLICATIONS, PROGNOSIS
○ boys (screamers) ● Phonotrauma ■ History ● Laryngoscopy ■ Conservative Management
VOCAL FOLD NODULES ○ women (talkative) ○ vocal abuse ● CC, HPI, ROS ○ bilateral
○ CHRONIC subepithelial fibrous thickening of the midpoint ○ professional voice users ○ inappropriate vocal use ● Vocal Overdoers ○ subepithelial fibrous thickening at midpoint of vocal fold ● Vocal Hygiene
of vocal folds due to vocal abuse & vocal misuse ○ talkativeness ○ occupation history – professional voice user □ junction between anterior 1/3 & posterior 1/3
● Contributing Factors ○ talkative – 6 or 7 on talkativeness scale Things to DO
■ Pathogenesis ○ allergy, LPR ○ child, ask if vocally exuberant
○ voice rest - briefly to prevent vocal fatigue, straining, and overuse
○ adequate hydration
○ forceful/prolonged vocal fold vibration ○ medications *aka “screamer’s nodules” ○ amplifier use - to reduce voice strain
→ localized vascular congestion w/ edema at midportion of ○ smoking, alcohol
○ indoor air humidifier - in dry, arid environments
Things to AVOID
membranous portion ○ hormonal ● Onset of Hoarseness ○ avoid irritants – smoking, secondhand smoking, drying medications
→ vibratory portion: where shearing & collisional forces ○ chronic hoarseness ○ limit large or spicy foods, limit caffeine – can dry, result in mucous thickening
○ avoid excessive throat clearing, coughing
are greatest ○ Hx repeated episodes of acute hoarseness ○ avoid overuse, straining voice – yelling, shouting, speaking over loud noises,
DIFFERENTIAL DIAGNOSIS: VOCAL FOLD LESIONS whispering
Clinical Laryngoscopy ○ onset correlated w/ URTI or acute laryngitis
Lesion Etiology
Manifestation □ after which, hoarseness didn’t heal complete
vocal abuse
● hoarseness
● bilateral
○ fibrovascular thickening at
● Vocal Therapy
Nodules female, 18-39 years
old
○ midline (junction bet ant ½ and
post 2/3) ● Identification of Abnormal Voice ○ education regarding behaviors & practices
vocal abuse, trauma, &
● hoarseness ● unilateral *more sensitive symptoms of vocal nodules ○ eliminate maladaptive behaviors
Cyst anticoagulant
(hemorrhagic cyst)
○ ○ encapsulated subepithelial mass
*from Cummings Otolaryngology 7th Ed *do laryngoscopy immediately if with alarm symptoms or if a ○ modify relevant speaker-specific & situation-specific
*VOCAL ABUSE: behavior that strains vocal folds (coughing, screaming) allergy, tobacco & ● unilateral, can be bilateral
*VOCAL MISUSE: improper usage (speaking too loudly, high pitch singing)
Polyp other irritants, vocal
● hoarseness

○ well-defined sessile or
professional voice user behaviors
abuse pedunculated midpoint mass
GERD, LPR, tobacco
& other irritants, vocal ● bilateral (commonly)
○ loss of ability to sing high notes softly *otherwise, do voice hygiene and voice therapy first; do
■ Histopathologic Evolution of Nodules Reinke’s abuse ● hoarseness ○ proliferation of superficial
○ delayed phonatory onset laryngoscopy only if FAILS TO IMPROVE in 2 WEEKS ■ Surgical Management
Edema female, middle-aged ○ frequent throat clearing mucosa over entire length of 1 or
○ incipient nodules are reversible (40-50) or older age
(>60yo)
both vocal folds
○ increased breathiness
○ basement membrane thickening, fibrosis, lamina propria traumatic mucosal
○ may be asymptomatic
● unilateral ○ reduced vocal endurance ● Laryngeal Videostroboscopy ● Microlaryngeal Surgery
edema Granuloma
disruption,
endotracheal
● hoarseness
○ discomfort at level of
○ hypertrophic inflammation,
exophytic with narrow base, usually ○ sensation of increased effort for singing ○ assess for mucosal waves ○ Options: Cold Knife, Laser Surgery
intubation, LPR
throat
○ dyspnea (if bulky)
in arytenoid region
○ need for longer warm ups ○ Indications:
HPV 6, 11; sexual
● hoarseness
○ airway obstruction ● asymmetric
□ reserved for symptomatic vocal fold lesions that are:
Papilloma transmission ○ stridor
○ weak cry (in babies)
○ benign cauliflower-like lesions
○ Hx intermittent hoarseness *severe, refractory to conservative management
Altered Voice
● hoarseness
○ low grade fever
● asymmetric Quality
○ progressive deterioration in HPI □ chronic nodules w/ ↓mucosal waves by stroboscopy
Tuberculosis MTB, close contacts ○ ulcerofungative, polypoid (breathy)
○ weight loss
○ night sweating ○ freuent pitch breaks
Pitch
smoking, excessive
● hoarseness – m/c
(glottic)
● asymmetric
○ reduced vocal range ● Post-Surgery: Refer to Speech Therapist
Laryngeal Loudness ○ voice fatigue (gets husky eventually)
SCCA
alcohol
FHx of malignancy
○ dysphagia (supreglottic)
○ dyspnea (subglottic)
○ friable, whitish plaque, any
portion, can extend contralaterally Voice Effort ○ sensation of increased effort
○ post-surgery, instruct the patient to do voice rest by not
HPV, LPR
○ (+) neck node
if at anterior commissure
speaking for 4 days
DIFFERENTIAL DIAGNOSIS: LARYNGITIS ○ beginning on 4th day, patient gradually progresses over 4
○ previous URTI ○ redness, thickening/edema of vocal cords ● Tertiary History
Acute Viral ○ hoarseness, mild sore throat ○ with viscous mucus coating
○ PMHx: allergy, LPR weeks to full voice under speech pathologist’s supervision
○ dry, nonproductive cough
Laryngitis ○ low grade fever
○ vocal straining ○ MedHx: exacerbating medications
○ foreign body (globus) sensation
○ frequent throat clearing
○ posterior laryngeal edema, pseudosulcus
○ diffused edema of true vocal folds
○ CHRF: smoking, alcohol
LPRD □ excessive phlegm in throat ○ posterior commissure hypertrophy
○ chronic cough ○ bilateral arytenoid swelling
○ hoarseness ○ thick endolaryngeal mucus
■ Physical Examination

● Vital Signs
○ fever - rule out viral laryngitis as cause
○ RR – check respiratory rate & pattern

● HEENT Exam
○ inspect & palate neck for masses

● Laryngoscopy – if indicated
*alarm symptoms, professional voice user

LACABA 20 | P A G E
UST MEDICINE – OTORHINOLARYNGOLOGY ORAL REVALIDA ΣΒΤΦ TITANS
DISEASE & PATHOPHYSIOLOGY EPIDEMIOLOGY ETIOLOGY CLINICAL MANIFESTATIONS DIAGNOSIS TREATMENT COMPLICATIONS, PROGNOSIS
*mainly viral ● History ● Laryngoscopy ● Eliminate and avoid exogenous irritants
ACUTE LARYNGITIS *bacterial superinfection may occur ○ hoarseness ○ redness, thickening/edema of vocal cords ○ avoid air pollution, smoking
○ inflammation limited to the larynx ○ dry, non-productive cough ○ with viscous mucus coating
○ usually assoc. with either an URTI or acute vocal strain ● Infectious Agents ○ mild sore throat ○ intralaryngeal mucosa may also be involved ● Voice Rest
○ Viral ○ low grade fever, malaise ○ avoid vocal fatigue
● Acute vs Chronic Laryngitis ○ Streptococcus pneumoniae ○ Hx vocal straining or URTI
○ acute laryngitis <3 weeks, self-limiting ○ Moraxella catarrhalis ○ NO signs of airway obstruction ● Mucolytic Agents
○ chronic laryngitis ≥3 weeks ○ Haemophilus influenzae □ dyspnea may occur in severe mucosal swelling ○ N-acetylcysteine 200mg/sachet 2-3x per day
□ dissolve 1 sachet in ½ glass of water

● Anti-inflammatory Agents – as needed for pain


○ as needed for pain
DIFFERENTIAL DIAGNOSIS ● Antibiotics – for bacterial superinfection
○ whitish glistening grayish-red mass in vocal cord ○ Amoxicillin 40-5- mkday in 3 divided doses x7 days
Vocal Fold Polyp
○ usually unilateral
Vocal Fold Nodules ○ whit mass bilaterally at midpoint of vocal fold

*multifactorial ● Symptoms ■ Reflux Scoring Minor Major Life Threatening

LARYNGEAL REFLUX DISEASE *UES relaxation ○ foreign body (globus) sensation SSx
annoying, but performance
impacts social life;
change in voice
airway obstruction, glottic &
subglottic stenosis,
is not impaired laryngospasm, paroxysmal
○ type of chronic laryngitis (≥3 weeks) ○ frequent throat clearing ● Reflux Symptom Index (Belafksy) (Subjective)
quality
fold involvement

○ d/t UES disorder (vs GERD which is a LES disorder) ● Refluxers □ excessive phlegm in the throat ○ based on history and clinical manifestations Tx
lifestyle modification
(eg avoid refluxers, reduce PPI for 3 months
PPI for 3-6 months;
surveillance every 2 months
□ retrograde movement of gastric contents ○ alcoholic beverage drinker ○ chronic cough ○ classify symptoms of patients into 9, each scored; alcohol, stop smoking)
(via repeat RSI, RSF -
endoscopy)
□ laryngopharynx is less able to tolerate gastric acids ○ coffee, cola, spicy foods ○ hoarseness □ 0 = no problem, 5 = severe problem
○ fatty & oily food ○ RSI > 13 is ABNORMAL! ■ Diet & Behavioral Modification
*caffeine causes UES relaxation ● Physical Exam
○ posterior laryngeal edema, pseudosulcus ● Behavioral Modification
● Other Risk Factors ○ diffused edema of the true vocal folds ○ voice rest: avoid vocal fatigue
○ obesity ○ posterior commissure hypertrophy ○ voice therapy: can help esp. if occupation depends on voice
○ smoking ○ bilateral arytenoid swelling ○ avoid vigorous exercise 2 hours after eating
○ thick endolaryngeal mucus ○ smoking cessation

● Diet Change
○ avoid alcohol, peppermint, caffeine, spicy, fatty foods
● Reflux Finding Score (More Objective) ○ increase hydration
○ based on laryngoscopic findings ○ maintain upright position 1-2hr after meals
○ divided into 8 findings
○ RSF > 7 = laryngopharyngeal reflux ■ Medical Therapy

Symptom Score ● Proton Pump Inhibitors – only if with GERD!


1 Subglottic edema (pseudosulcus) 0 = absent, 2 = present
○ Omeprazole 40mg/tab 1 tablet OD
2 Ventricular obliteration 0 = none, 2 = partial, 4 = complete

3 Erythema/hyperemia
0 = none, 2 = arytenoids only, 4 = □ 30 mins before breakfast for 6-8 weeks
diffuse
0 = none, 1 = mild, 2 = moderate, 3 = □ OR at least 2 weeks as empirical treatment
4 Vocal fold edema severe, 4 = polypoid
0 = none, 1 = mild, 2 = moderate, 3 =
*if improving → taper down the dose
5 Diffuse laryngeal edema severe, 4 = obstructing
0 = none, 1 = mild, 2 = moderate, 3 =
*if no response → increase dose, for 6-8 more weeks
6 Posterior commissure hypertrophy severe, 4 = obstructing ○ no benefit if without concomitant GERD
7 Granuloma/Granulation 0 = absent, 2 = present
8 Thick endolaryngeal mucus 0 = absent, 2 = present
■ Reflux Surgery
■ Diagnostic Modalities
● Fundoplication
DIFFERENTIAL DIAGNOSIS ● Trial of PPI Therapy ○ those who DON’T response to behavioral & medical Tx
○ hoarseness, mild sore throat
○ dry, nonproductive cough ● Esophageal Manometry
Laryngitis ○ usually as a guide for placement of pH probe
○ low grade fever
○ vocal straining ○ measures: length, sphincter pressure, assess motility of
GERD LPRD body of esophagus
WITH esophagitis ± heartburn RARELY esophagitis ± heartburn
reflux is NOCTURNAL reflux is DAY time ● 24hrs pH Monitoring - GOLD STANDARD
reflux in SUPINE reflux in UPRIGHT ○ but not frequently done due to cause
d/r prolonged acid exposure intermittent reflux episodes ○ quantify actual time mucosa is exposed to gastric juice
LES dysfunction UES dysfunction ○ measures ability of esophagus to clear refluxed acid
esophageal symptoms throat-related symptoms ○ positive for reflux: decrease in pH level <4.0 or 5.0
heartburn, acid regurgitation, chest throat clearing, chronic cough,
pain hoarseness, excessive phlegm in ● Laryngeal Biopsy
throat, globus sensation ○ similar to esophageal biopsy of GERD
○ dilation of intercellular spaces
CONSIDER DIFFERENTIALS FOR CHRONIC COUGH
○ numerous cytoplasmic vacuoles

LACABA 21 | P A G E
UST MEDICINE – OTORHINOLARYNGOLOGY ORAL REVALIDA ΣΒΤΦ TITANS

HEAD & NECK MASSES


Approach to HEAD & NECK MASSES
NECK MASS DDx CC: ANTERIOR NECK MASS ■ Physical Examination MANAGEMENT APPROACH TO NECK MASSES
■ Generalities in Assessing Neck Masses ■ Common Differential Diagnosis for REVALIDA *also check for signs of hyperthyroidism and hypothyroidism! ■ Management According to Duration of Mass
*there are 4 questions in determining the most probable differential diagnosis for neck mass *differentials obtained from Otorhinolaryngology Batch 2021 Lectures
● Vital Signs
(1) Mass Type: inflammatory, congenital, neoplastic, traumatic ● Anterior Midline Neck Mass ○ check for fever → reactive lymphadenitis, acute bacterial lymphadenitis, infected congenital cysts
□ to know what the most probably type of mass, ask 2 subsequent questions: ○ Adenitis
□ What is the location? How old is the patient? ○ Thyroglossal duct cyst – case in oral revalida ● Skin Examination
○ Laryngocoele ○ check for exanthems → reactive lymphadenitis
(2) Location: where is the mass? anterior midline, anterior triangle, posterior triangle ○ Dermoid Cyst
○ Benign Thyroid: Grave’s Disease; Endemic Goiter; Acute, Subacute, Painless, Reidel’s Thyroiditis ● Eye Examination
Location Anterior Midline Anterior Triangle Posterior Triangle ○ Malignant Thyroid: Papillary, Follicular, Medullary, Anaplastic ○ ophthalmoplegia, proptosis → Grave’s disease
○ Adenitis ○ lid lag → hyperthyroidism
Inflammatory ○ Adenitis ○ Adenitis
○ Sialadenopathy
● Anterior Triangle Neck Mass
○ Thyroglossal duct cyst ○ Branchial Cyst
○ Adenitis ● Head & Neck Examination
Congenital ○ Dermoid cyst ○ Thymic Cyst ○ Lymphangioma ○ Head & Face
○ Laryngocoele ○ Sialadenopathy ○ Branchial Cleft Cyst
○ Neoplastic Adenopathy ○ TB Lymphadenitis □ inspect for facial symmetry, color, deformities → asymmetry may mean malignant tumor
○ Neoplastic Adenopathy □ perform mass examination (see below)
□ Metastasis Level I-IV ○ Metastatic lymph nodes
○ Thyroid □ Metastasis Level I-IV
Neoplastic
○ Lymphoma
□ Lymphoma
□ Lymphoma □ nasopharyngeal, oral, oropharyngeal cancers *inspect for location, size, overlying skin
○ Primary vascular tumors *palpate for size, surface, shape, consistency, borders, mobility
○ Neuroma □ Virchow (GI), Supraclavicular (Breast CA)
○ Sialadenopathy
*auscultate for bruits
Traumatic ○ SCM Hematoma ○ False aneurysm
■ History ○ Ear Examination
□ patient has neck mass, may extend such and predispose to middle ear manifestations etc.
(3) Age: what is the age of the patient? pediatric (≤15yo), young adult (≤40yo), or older adult (>40yo)?
● General Data ○ Mouth, Throat Examination
□ the most common type of neck mass differs depending on the age of the patient
○ age – very important! □ inspect for oral mass/cancers, especially if neck lymphadenopathy suggests drainage from this area ■ Imaging of Choice Depending on Clinical Scenario
□ if pediatric or younger adult, most common causes are LYMPHADENITIS so check for infection □ inspect for enanthems, may be reason for ○ done when you have a subacute or chronic mass
Age Pediatric (≤15yo) Young Adult (≤40yo) Older Adult (>40yo)
Most Common Inflammatory Inflammatory Neoplastic □ if older adult, most common causes are NEOPLASTIC so check for risk factors and evolution of mass ○ Neck Exam ○ also done when an acute mass does not resolve after 2 weeks of treatment
Congenital Congenital Inflammatory ○ geography/ancestry – if considering NPCA which is more common in southeast china □ inspect: ○ if (+) Hx of CA treatment: do a PET Scan to see picture of the whole body, check metastasis
↓ *symmetry, color, deformities, contour defining structures, changes in overlying skin
Neoplastic Neoplastic Congenital
Least Common Traumatic Traumatic Traumatic
● CC, HPI, ROS *fistulous openings (branchial fistula, thryoglossal fistula, actinomycosis)
○ where is the mass – have the patient point to where the mass is (with inspection PE) □ palpate:
(4) Duration: how long has the mass been present? acute, subacute, or chronic *lifted from AAFP 2015 ○ IS THERE PREVIOUS TRAUMA? – to rule out traumatic causes! *preauricular, postauricular, submandibular, submental, jugulodigastric, cervical, supraclavicular
○ onset of mass – is it acute, subacute, or chronic? *check for Virchow’s Node (GI CA), check for supracervical lymphadenopathy (breast CA?)
Duration Acute Subacute (weeks-months) Chronic (years)
□ if acute: think adenitis or trauma □ if mass is present, do mass examination
○ Reactive lymphadenitis
□ if subacute: cancers eg upper aerodigestive SCCA, parotid tumor, metastatic tumor (Virchow’s) □ perform thyroid mass examination
□ CMV, EBV, viral URTI
○ SCCA of the upper □ if chronic: think thyroid, congenital (thyroglossal, dermoid, branchial), laryngocele, lipoma, liposarcoma
Common ○ Acute Bacterial ○ Thyroid pathology
lymphadenitis
aerodigestive tract
○ differential: acute bacterial/reactive lymphadenitis ● Mass Examination – don’t forget to assess for the 10 CHARACTERISTICS OF A MASS!
□ Staph, Strep
□ ask for SSx infection: fever, malaise 01 Location anterior midline, anterior triangle, posterior triangle
○ Acute Sialadenitis 02 Size inspect and palpate the size of the mass
○ TB Lymphadenitis
○ Metastatic cancer ○ Thyroglossal duct cyst □ recent URTI?
○ Cat scratch disease
○ Parotid Tumor ○ Branchial cleft cyst ○ differential: cancerous tumor 03 Surface smooth, rough, lobulated
○ Lymphoma ○ Laryngocele
Uncommon (Bartonella)
○ Amyloidosis ○ Lipoma
□ rapid weight loss, anorexia, cachexia 04 Shape rounded, ovoid, etc.
○ HIV infection ○ differential: TB Lymphadenitis inspect color of overlying skin: erythematous
○ Sarcoidosis ○ Carotid Body Tumor
○ Pseudoaneurysm 05 Skin
○ Hematoma
○ Sjogren Syndrome ○ Glomus tumor □ SSx PTB: chronic cough, night sweats, weight loss skin mobility over the mass: indurated? Malignant tumor
○ Castleman disease 06 Consistency firm, soft, doughy, hard, rubbery fluctuant?
○ Kikuchi disease ○ Liposarcoma ● ROS 07 Tenderness palpate the mass: is it tender → acute infectious sialadenitis
Rare -
○ Kimura disease ○ Parathyroid carcnoma ○ rashes (exanthems) diffuse? think non-tumors
○ Rosai-Dorfman disease
○ ear (AOM, COM): otalgia, otorrhea, hearing loss 08 Borders
circumscribed? think tumors → assess if ill-defined or well-defined!
○ nose (nasopharyngeal mass): nasal obstruction, epistaxis 09 Mobility Is the mass movable or fixed? If fixed → infiltrating malignant tumors!
○ SSx hyperthyroidism: weight loss despite ↑ appetite, heat intolerance & sweating, hyperdefecation,
10 Bruit Check for bruit, esp if thyroid mass → Grave’s Disease
palpitations, tremors, AUB
○ SSx hypothyroidism: weight gain, cold intolerance, constipation, difficulty concentrating, poor memory, AUB
● Thyroid Examination - also perform mass examination on thyroid masses
○ obstructive symptoms: dyspnea, dysphagia, dysphonia
○ Inspection
○ nasal obstruction – if thinking possible nasopharyngeal tumors
□ WHO Goiter Grading (see below)
□ size, gross shape and symmetry, overlying skin (erythema?)
PMHx
○ middle ear infections (AOM, COM) – suggests nasopharyngeal mass lesions (block eustachian tube) □ movement on deglutition – positive for both TGDC and thyroid
○ previous EBV infection (NPCA) □ movement on tongue protrusion – positive only for TGDC
○ previous neck irradiation ○ Palpation
○ history of trauma – to rule out traumatic causes!!! □ tenderness: non-tender, exquisitely tender
□ enlargement: diffuse, nodular, multinodular?
● Current Health & Risk Factors □ consistency: firm, rubbery, stony hard
○ Percussion
*risk factors for malignancies of head & neck eg oral cavity, salivary, nasopharyngeal cancers
□ retrosternal dullness
○ risk factor for oral cavity tumors: smoking, alcohol, UVB
○ Auscultation
○ illicit drug use – marijuana (NPCA)
○ eating habits – salted/smoked fish, preservatives (NPCA) □ bruits – grave’s disease!

Grade PE Findings
0 no goiter
Ia palpable only
Ib visible when neck is fully extended
II visible in normal position
III visible at a distance

LACABA 22 | P A G E
UST MEDICINE – OTORHINOLARYNGOLOGY ORAL REVALIDA ΣΒΤΦ TITANS
Approach to HEAD & NECK MASSES
CC: SALIVARY GLAND MASS/SWELLING ■ Physical Examination
■ Common Differential Diagnosis for REVALIDA
● Vital Signs
● Acute Salivary Gland Swelling ○ check for fever → acute infectious sialadenitis
○ Sialolithiasis
○ Acute Viral Sialadenitis (Mumps) ● Skin Examination
○ Acute Bacterial Sialadenitis (Suppurative) ○ check for exanthems → mumps, coxsackie

● Chronic Salivary Gland Swelling ● Head & Neck Examination


○ Sialadenosis ○ Head & Face
○ Chronic Sialadenitis: Myoepithelial (Immune), Radiation □ inspect for facial symmetry, color, deformities → asymmetry may mean malignant tumor
○ Pleomorphic Adenoma – most common benign salivary gland tumor □ perform mass examination (see below)
○ Mucoepidermoid Carcinoma – most common malignant salivary gland tumor *inspect for location, size, overlying skin
*palpate for size, surface, shape, consistency, borders, mobility
● Non-Salivary Gland Differentials *auscultate for bruits
○ cervical lymphadenitis ○ Ear Examination
□ patient has neck mass, may extend such and predispose to middle ear manifestations etc.
○ Mouth, Throat Examination
□ denture use? check esp if differential is sialolithiasis
□ check ductal orifice: erythematous? spontaneous pus drainage? turbidity? absent salivary flow?
*if no pus drainage & acute bacterial sialadenitis is suspected, massage the gland & check if express pus
○ Neck Exam
□ inspect: symmetry, color, deformities
□ palpate: preauricular, postauricular, submandibular, submental, jugulodigastric, cervical, supraclavicular
□ if mass is present, do mass examination

● Mass Examination – don’t forget to assess for the 10 CHARACTERISTICS OF A MASS!


01 Location inspect: where is the mass? Parotid or submandibular or others?
02 Size inspect and palpate the size of the mass
03 Surface smooth, rough, lobulated
04 Shape rounded, ovoid, etc.
inspect color of overlying skin: erythematous
05 Skin
skin mobility over the mass: indurated? Malignant tumor
06 Consistency firm, soft, doughy, hard, rubbery fluctuant?
07 Tenderness palpate the mass: is it tender → acute infectious sialadenitis
diffuse? think non-tumors
08 Borders
circumscribed? think tumors → assess if ill-defined or well-defined!
09 Mobility Is the mass movable or fixed? If fixed → infiltrating malignant tumors!
10 Bruit
Benign Salivary Gland Tumor Malignant Salivary Gland Tumor
■ Diagnostic Approach for SALIVARY GLAND NEOPLASMS
○ slow growth (months to years) ○ rapid growth (weeks to months)
○ painless, soft or tense nodule, freely movable ○ painful, fixed nodule ● Diagnostic Imaging – done ONLY in rare/deep tumors
○ no signs of tumor infiltration ○ tumor infiltration: facial nerve palsy, pain ○ CT and MRI do NOT differentiate benign from malignant
○ no additional symptoms ○ lymphadenopathy ○ value: assess local extent, invasion, determine resectability, assess for non-palpable neck LN
○ MRI: most sensitive, diagnostic imaging of choice
■ History ○ CT: done only if with fixation to underlying structures (skin induration, bone, facial nerve palsy)
● General Data ● Preoperative Biopsy
○ age – benign is common in younger; malignant is common in 5th-6th decades ○ FNAB is the biopsy of choice but is NOT routine
□ clinically benign? no need for FNAB → surgical resection regardless of FNAB results
● CC, HPI: Salivary Gland Mass □ clinically malignant? do FNAB to confirm → see below
*ask these 4 basic questions to follow the clinical algorithm ○ In clinically malignant, FNAB helps the following:
*the 5th component (diffuse vs circumscribed) is assessed during mass physical examination □ is this metastatic or primary neoplasm?
○ timing: acute or chronic swelling? *if primary, remove the gland
□ if acute (days to weeks), think infectious causes (tenderness, fever, pain) & sialolithiasis *if secondary, look for primary metastasis
□ if chronic (months to years), think of sialadenosis, chronic inflammations, and tumors □ is this malignant neoplasm surgically treatable?
○ pain: present or absent? If present, continuous or increases with food? *if lymphoma: has many lesions, treat with chemo and radiothrapy
□ if painful, usually this is an acute process
*ask for aggravating factors, if patient took anything for the pain ■ Treatment Approach for SALIVARY GLAND NEOPLASMS (NCCN 2021)
□ if painless, think of chronic causes
*keep in mind, malignant tumors present eventually with pain d/t nerve impingement ● Determine Primary Tumor designation
○ localization: parotid gland or submandibular gland? T1 ≤ 2 cm
○ laterality: bilateral or unilateral? T2 ≤ 4cm
□ acute bilateral → usually acute viral (others may or may not by bilateral) T3 > 4cm and/or extraparenchymal extension: clinical or macroscopic evidence of soft tissue invasion
□ acute unilateral → acute bacterial, sialolithiasis T4a Moderately Advanced Disease invades skin, mandible, ear canal, and/or facial nerve
□ chronic bilateral → sialadenosis, immune sialadenitis T4b Very Advanced Disease invades skill base and/pr pterygoid plates and/or encases carotid artery
□ chronic unilateral → tumors if circumscribed
● Clinically Benign or T1, T2 Lesions
● ROS ○ complete resection with intraoperative frozen section histology
○ Mouth: Xerostomia? Ageusia? ○ depending on pathology result:
○ SSx rheumatologic diseases (arthralgia, skin changes) □ benign → follow-up
○ SSx DM (3P), hypothyroid (deep voice, slowed mentation, cold intolerance) □ low-grade with tumor spillage or perineural invasion → consider adjuvant RT
□ adenoid cystic, intermediate or high grade → adjuvant RT
● PMHx
○ risk factors for acute bacterial sialadenitis: DM, hypothyroidism, renal failure, Sjogren’s syndrome ● T3, T4a Lesions
○ risk factors for chronic sialadenitis ○ assess if for surgery or not
□ Sjogren, RA, SLE, Polymyositis, Scleroderma → Immune Myoepithelial sialadenitis ○ if NOT for surgery → definitive RT or concurrent systemic + radiotherapy
□ previous irradiation, radioiodine therapy → Radiation Sialadenitis ○ if for surgery → determine cancer site, clinical node positivity, tumor grade

Gland Clinical Node Treatment


parotidectomy with complete tumor resection
N0
Parotid ± neck dissection (for high grade and high stage tumors)
N+ parotidectomy + neck dissection
N0 complete tumor resection
Others
N+ complete tumor resection + lymph node dissection

● T4b Lesions → definitive RT or concurrent systemic + radiotherapy

LACABA 23 | P A G E
UST MEDICINE – OTORHINOLARYNGOLOGY ORAL REVALIDA ΣΒΤΦ TITANS
ACUTE SALIVARY GLAND SWELLING

DISEASE & PATHOPHYSIOLOGY EPIDEMIOLOGY ETIOLOGY CLINICAL MANIFESTATIONS DIAGNOSIS TREATMENT COMPLICATIONS, PROGNOSIS
*eating incites swelling and severe pain!
○ adults; 3rd & 4th decade *acute bacterial sialadenitis is a complication but ● Ultrasound ● Submandibular Gland Stones ■ Complications
SIALOLITHIASIS ○ males (2:1) may present as a primary or secondary symptom ○ dilatation of duct system ○ distal stone ○ stasis-induced “salivary tumor”
○ stone formation in excretory duct system of salivary gland ○ echogenic sialolith with acoustic shadowing (70%) □ marsupialization w/ stone extraction ○ infection and abscess if the oral floor
○ microcalcification of mucins lead to salivary stasis ● Salivary Gland ● Salivary Gland Swelling ○ intraductal stones ○ stasis-related bacterial infection of the excretory duct
○ submandibular (70-80%) Timing Acute ● Oral Floor Radiograph □ endoscopic fragmentation and gland → Acute Sialadenitis
○ parotid (20%) Laterality Unilateral or Bilateral ○ demonstrate distal Wharton’s duct stones □ extracorporeal lithotripsy
Pain Present, Increases with eating
Swelling Increases with eating ○ only in rare cases when UTZ is equivocal ○ intraglandular stones
Others ○ consistency: doughy □ excision of chronically damaged gland
DIFFERENTIAL DIAGNOSIS ○ duct orifice: look for obstructing material *parotid gland calculi are difficult to define on radiograph:
○ palpable stones in the duct (submandibular)
Other Causes of Salivary Gland Duct Obstruction ○ tenderness on palpation
poorly calcified sialoliths, obscured by superimposed ● Parotid Gland Stones
Dentures ○ ask about denture use structures ○ conservative therapy – tried first
Tumors
○ chronic palpable, nontender mass □ analgesics, increased fluid intake
○ facial nerve palsy ○ parotidectomy – rarely indicated

● Mumps (rubulavirus) – m/c ● Salivary Gland Swelling - “hamster cheeks” ○ diagnosis is clinical *should resolve in 1-2 weeks ■ Complications
ACUTE VIRAL SIALADENITIS ○ MOT: droplet, shed in saliva Timing Acute ○ serologic testing – only in doubtful cases
○ aka epidemic parotitis, infectious parotitis ○ hematogenous to salivary glands Laterality Bilateral □ 4x rise in Ab 2-3 weeks after onset of disease ● Supportive ● Common Complications
○ acute viral inflammation of the salivary glands Pain Present, Constant
○ analgesics ○ serous meningitis
Swelling Diffuse
● Other rare causes Others ○ consistency: doughy edema ○ increase fluid intake
○ CMV ○ duct orifice: reddened, slightly swollen ○ salivary stimulation – use lemon drops ● Serous, Less Frequent Complications
○ Coxsackievirus ○ meningoencephalitis with permanent CN defects
○ Echovirus ● Other Symptoms ● Preventive ○ orchitis, ovaries, pancreas
○ Parainfluenza virus ○ mild fever (70%) ○ MMR vaccine - 6, 10, 14 weeks ○ labyrinthitis
○ Influenza virus ○ swelling of cervical lymph nodes ○ unilateral hearing loss
○ HIV

○ usually in debilitated, ● Etiology ● Salivary Gland Swelling ○ diagnosis is clinical ● Medical Management ● Abscess
ACUTE BACTERIAL SIALADENITIS dehydrated patients ○ S. aureus – main agent Timing Acute □ suppurative discharge ○ antibiotics ○ Tx: incise parotid gland, parallel to branches of facial
○ aka acute suppurative sialadenitis ○ Streptococci Laterality Unilateral ○ culture & sensitivity of pus ○ analgesics (NSAIDs) nerve
○ acute bacterial inflammation of the salivary glands ○ Haemophilus
Pain Present, Constant
○ hydration
Swelling Diffuse
Others ○ skin over gland: is reddened, may be ○ salivary glands stimulation
● Risk Factors fluctuant (d/t tissue liquefaction) ○ good oral hygiene
○ duct orifice: red & swollen; turbid fluid/pus
○ DM, hypothyroidism, renal failure, expressed or drain spontaneously
DIFFERENTIAL DIAGNOSIS Sjogren’s syndrome
Dentogenic ○ can produce similar findings
○ Sialolithiasis
Infections ● Other Symptoms
Furuncular Otitis with periauricular abscess or lymphadenitis ○ trismus

CHRONIC SALIVARY GLAND SWELLING

DISEASE & PATHOPHYSIOLOGY EPIDEMIOLOGY ETIOLOGY CLINICAL MANIFESTATIONS DIAGNOSIS TREATMENT COMPLICATIONS, PROGNOSIS
○ women ○ painless, firm, nodular salivary gland ● Clinical diagnosis ● Complete Tumor Resection ● Prognosis
PLEOMORPHIC ADENOMA ○ freely movable ○ painless, soft or tense, mobile nodule *Lateral parotidectomy for parotid gland ○ prognosis is good following sound tumor resection
○ most common BENIGN TUMOR of salivary glands ● Salivary Gland ○ usually confined to one side ○ without any other symptoms ○ most common procedure done for benign salivary gland ○ smaller tumors: prompt resection since this is easier to
○ most common adenoma of salivary gland ○ m/c in parotid gland tumors remove and preserve facial nerve
● Features of BENIGN TUMOR ● Biopsy ○ complete removal with margin of healthy tissue
○ slow growth (months to years) ○ done WITH treatment, done in one step ● Complications
○ painless, soft or tense nodule, freely movable
○ no signs of tumor infiltration ○ intraoperative frozen section of tissue histology after *intraoperative frozen tissue histology of surgical specimen for ○ carcinoma may occur but is RARE
○ no additional symptoms surgery definitive pathologic diagnosis

*incisional biopsy is NOT DONE due to risk of tumor cell


dissemination and facial nerve injury

○ can be in YOUNG ○ initially: non-painful swelling ● FNAB ● Surgery with Post-op Radiotherapy ● Prognosis
MUCOEPIDERMOID CARCINOMA ○ later manifestations – depending on tumor grade ○ confirm suspicion of malignant tumor on HxPE ○ complete tumor resection with R0 margins ○ depends strongly on GRADE
○ most common MALIGNANT TUMOR of salivary glands ● Salivary Gland □ pain □ total parotidectomy ± facial nerve preservation ○ well-differentiated low-grade tumors
○ parotid gland – □ facial nerve palsy ● MRI or CT Scan ○ postop adjuvant radiation □ 5-year survival rate of 90%!
prodimnantly □ lymph node metastasis ○ done if suspected malignant to: *depending on extent of the tumor, may include SUBTOTAL removal of
○ minor salivary glands of □ define extent of tumor temporal bone, mandible, skin, vessels, and/or cervical lymph nodes ● Complications
the palate ● Features of MALIGNANT TUMOR □ invasion of adjacent structures ○ metastasis usually via lymphogenous route
○ rapid growth (weeks to months) ● Reconstructive Surgery
○ painful, fixed nodule ○ facial nerve - nerve grafting
○ tumor infiltration: facial nerve palsy, pain ● Frozen Tissue Histology
○ done ○ ICA - bypass or replacement graft
○ lymphadenopathy
○ skin – advancement flaps

LACABA 24 | P A G E
UST MEDICINE – OTORHINOLARYNGOLOGY ORAL REVALIDA ΣΒΤΦ TITANS
ANTERIOR NECK MASS

DISEASE & PATHOPHYSIOLOGY EPIDEMIOLOGY ETIOLOGY CLINICAL MANIFESTATIONS DIAGNOSIS TREATMENT COMPLICATIONS, PROGNOSIS
○ most common congenital ● History ● Clinical Diagnosis *simple excision is associated with HIGH RECURRENCE ● Infected TGDC
THYROGLOSSAL DUCT CYST neck cyst ○ chronic anterior midline neck mass ○ inspection & palpation RATE (45-55%) ○ most have some degree if inflammation at presentation
○ cyst of epithelial remnants of the thyroglossal tract due to ○ 75% manifest before 5yo ○ ETIO: oropharyngeal flora (strep, oral anaerobes)
failure of closure of the thyroglossal duct ○ most diagnosed <1yo ● Physical Examination ● Ultrasound – ideal initial imaging of choice ● Sistrunk Operation ○
○ Inspection ○ well-circumscribed, elliptical ○ STANDARD surgical management
■ Embryology □ midline neck mass, between chin & thyroid ○ hypoechoic or echo-free mass ○ resection of the cyst PLUS:
○ 3 weeks AOG: thyroid forms □ overlying skin: normal ○ (+) distal acoustic enhancement □ central portion (body) of the hyoid bone
□ at junction bet ant 2/3 and pos1/3 of the tongue *erythematous when infected *also evaluate presence of healthy thyroid tissue! □ and tissue surrounding the thyroglossal tract
□ location of foramen cecum □ moves with deglutition – like thyroid
○ thyroid descends to the neck, anterior to the hyoid □ elevate with tongue protrusion – unlike thyroid ■ Infected TGDC
○ reaches final position at 7 weeks AOG ○ Palpation *AVOID surgery when the TGDC is inflamed!
□ soft and cystic *manage initially with antibiotics, followed by elective Sistrunk
surgery once infection has resolved
DIFFERENTIAL DIAGNOSIS
*note: always look for formed thyroid
○ more lateral in location
Branchial Cleft Cyst
○ NOT related to the hyoid bone *note: these can be infected and may present as an ● Antibiotics – ONLY IF INFECTED!
○ may be anterior midline mass inflammatory anterior neck mass ○ MILD
Dermoid Cyst ○ NOT related to the hyoid bone, may be □ Amoxicillin-Clavulanate 500mg/125mg PO q8 hours
located at the thyroid (lower than TGDC) □ Cephalexin 500mg PO q6 hours
Cystic Metastatic LN ○ associated papillary thyroid CA, SCCA □ Clindamycin 600mh PO q8 hours
○ moves with deglutition ○ SEVERE
Goiter ○ may be hyper/hypothyroid
○ firm, doughy; NOT cystic □ Cefazolin + Clindamycin

LACABA 25 | P A G E

You might also like